Вы находитесь на странице: 1из 107

Professional Responsibility

CHAPTER 1 INTRODUCTION TO LEGAL ETHICS Lecture: Importance of being professionals Discussion Problems Apply Rules: 1.2d, 1.6, 1.7(a)(b), 4.4(b), 1.14(b) 1) Your client is the plaintiff in civil action against five corporate defendants. The case is ready for trial. You receive a fax. The cover page indicates the document was sent by one of the defendants attorney and was intended for the other four defense attorneys. Apparently the document was sent to you by mistake. There is a standard boilerplate statement on the cover page that this is a confidential communication protected by attorney-client privilege, work product doctrine, or both. The opening on the first page indicates that the memo states the senders clients position on settlement and outlines the negotiating tactics the attorney believes should be used to negotiate with you. What should you do? Choices: Read it and use it Do not read it and let the other side know you received Do not read it, but hold onto it without telling the other side Read it and withdraw as attorney for plaintiff because now unqualified Utilitarian Ask which option would produce the greatest number and displease the fewest number of people. What effect on the judicial system will our action have? What about respect for the rule of law and the court system? Golden Rule Return the document to the defense attorney unread Categorical Imperative Take the course of conduct that can be stated as a maxim/cannon that can be followed. Will probably arrive at the same result under all three approaches Rule 4.4(b) A lawyer who receives a document relating to the representation of the lawyers client and knows or reasonable should know that the document was inadvertently sent shall promptly notify the sender. Comment 3 may chose instead to return the document unread Has sender waived attorney-client privilege and any claim of work product? Some courts say yes. Some say no and disqualify the attorney if he read the document. 1|Page

a violation of the rules of prof. conduct involves a disciplinary action; contrast with a violation of attorney-client privilege or a violation of the rules of the court in this cases, the defendant should ask that the attorney be disqualified from the case Moral: Make sure those in your office know the importance of being accurate in this regard. 2) For years you have served as legal counsel for a family in a variety of matters. Mother and father have now died. Son and daughter each inherit $750,000. Son in unmarried but has an 11-year old child who lives with him. You have represented son in various matters. You have concluded son has a drug problem and is unfit to take care of himself, his estate, and his child. The sister asks you to petition the court to appoint a conservator (responsible for the estate like a guardian would be for a person) for the son. You know the son will adamantly oppose the idea. What should you do? Four people with interests: Samuel needs but doesnt want a conservator Dana wants what is best for Samuel Clara would want estate preserved Attorney we care about our client; we have an interest in fees (if we file petition we may be able to represent the estate in the future) You may represent Dana or not. Cannot use any information you learned from Samuel against him. Rule 1.14(b) When the lawyer reasonably believes that the client has diminished capacity, is at risk of substantial physical, financial or other harm unless action is taken and cannot adequately act in the clients own interest, the lawyer may take reasonably necessary protective action, including consulting with individuals or entities that have the ability to take action to protect the client and, in appropriate cases, seeking the appointment of a guardian ad litem, conservator or guardian. May = It is not required, but you can do so. Rule 1.7(a)(1)(2) a lawyer shall not represent a client if the representation involves a concurrent conflict of interest. A concurrent conflict of interest exists if the representation of one client will be directly adverse to another client; or there is a significant risk that the representation of one or more clients will be materially limited by the lawyers responsibilities to another client, a former client or a third person or by a personal interest of the lawyer. Rule 1.6(a) A lawyer shall not reveal information relating to the representation of a client unless the client gives informed consent, the disclosure is impliedly authorized in order to carry out the representation, or the disclosure is permitted by paragraph (b). Rule 1.6(b)(1) A lawyer may reveal information relating to the representation of a client to the extent the lawyer reasonable believes necessary to prevent reasonably certain death or substantial bodily harm. Comment 17 The duty of confidentiality continues after the client-lawyer relationship has terminated. 2|Page

May 16, 2010 (first day) Issue dont want to breach confidentiality to Samuel, doesnt want to go against Samuels wishes, conflict of interest b/t Dana and Samuel, concern for personal attorney reputation of interfering with the clients affair. How does the ABA address the issue? R. 1.14 (drastically changes in 2002). See Rule! Give permission to take minimal (reasonable) to take action to protect clients interest. (See also 1.6(a) when taking protective action pursuant to 1.14(b), implied under 1.6(a) if only to the extent reasonably necessary to protect the clients interests. (Be careful make sure your jurisdiction follows). 3) Three years ago Leon represented Curtis in a workers compensation matter. There was a settlement agreement and a check from the insurance company comes every month to Leon. Leon deducts 10% as his legal fee and remits the remainder to Curtis. During a routine review of the file Leon becomes aware the insurance company has made a mistake and has been sending almost twice the amount Curtis is entitled to receive. Leon informs Curtis. Curtis tells Leon he needs the money and not to rock the boat. What should Leon do? May 16, 2011 First thingtalk to your client about the legal ramifications if keep the money (larceny etc.) Whats bothering you as a lawyer? Dishonesty, loyalty, confidentiality, cant act against your clients interest unless the law requires (when does the law requires R 1.16, 2.1, 1.2(d)(A), 1.16(b)(2)(May (dont have to) only if your services is being used to furtherance the crime). Cts split but lean to Rule, if disclosing would reveal past crimes cant reveal. Be certain of the facts! Recalculate! Then counsel the client about these circumstances. Advise the client that he has civil liability if he has received money erroneously and the insurance company has a civil claim against him. There could be criminal charges brought for future payments and this would be a felony. Suggest you contact the insurance company and work out a way to repay the funds over time by a deduction from future payments. If he refuses to allow this, you cannot assist him in carrying out a fraud or crime and must withdraw. Give notice to insurance company that you have withdrawn and take yourself out of the loop. Cannot reveal the fraud. What do we do about the money we have received? We owe the money back, but we have a duty not to disclose information regarding the representation of the client. There is no answer to this in the rules. Best solution is to return the money without revealing the source. Best way to do this is to return the money anonymously through counsel of your own. (Brown Paper Bag) This is a situation when theres a need for good hard lawyering--- you need to advise Curtis that the law does not

3|Page

permit him to retain these funds in error. This is where you have to truly be a counselor of the law. Rule 2.1 (Advisor): In representing a client, a lawyer shall exercise independent professional judgment and render candid advice. In rendering advice, a lawyer may refer not only to law but to other considerations such as moral, economic, social and political factors that may be relevant to the clients situation. Rule 1.2(d) A lawyer shall not counsel to engage, or assist a client, in conduct that the lawyer knows is criminal or fraudulent, but a lawyer may discuss the legal consequences of any proposed course of conduct with a client and may counsel or assist a client to make a good faith effort to determine the validity, scope, meaning or application of the law. Rule 1.6(a) A lawyer shall not reveal information relating to the representation of a client unless the client gives informed consent, the disclosure is impliedly authorized in order to carry out the representation, or the disclosure is permitted by paragraph (b). Rule 1.16(a)(1) (must withdraw)

Chapter two May 16, 2011 (Big Changes were 5.5 and considering the adoption of Universal bar Exam, 12 states have already adopted) The Relationship Between Ethics and Legal Ethics Ethics = moral philosophy Legal Ethics = the rules or standards governing the conduct of members of the legal profession Utilitarianism Actions are right or wrong by virtue of their consequences All that matters in assessing consequences is the amount of happiness or unhappiness caused right actions produce the greatest balance of happiness over unhappiness (Greatest Happiness Principle) Hedonism No ones happiness is more important than anyone elses. Each persons welfare is equally important. The Golden Rule Do to others as you would have them do to you The Categorical Imperative Theory of human obligations set of principles of obligations The focus is on action rather than either results or entitlements. We should look at the nature of the proposed actions themselves. The Formula of the End in Itself Act in such a way that you always treat humanity, whether in your own person or in the person of any other, never simply as a means but always at the same time as an end. We use others as mere means if what we do reflects some maxim to which they could not in principle consent (because there was deception or coercion) 4|Page

CHAPTER 2 SOURCES AND APPLICATION OF LEGAL ETHICS RULES Discussion Problems 1) Under the laws of the state in which you plan to practice law, what requirements will you have to meet to be admitted to practice? Must be at least 19 years old (Ohio 21) Must be a graduate of an ABA approved law school or Miles, Bham School of Law, or Jones School of Law; graduates of other law schools must get a waiver from the Alabama Supreme Court Must pass the Bar Exam may take it an unlimited number of times must pay application fee Bar Exam is administered by the AL State Bar under the AL Supreme Court o In the past, AL had the diploma privilege automatically admitted to the bar upon graduation from AL School of Law o Some other states have reciprocity AL and most states do not Must be of good moral character character and fitness must be approved by the Committee on Character and Fitness (work in panels of three) must have affidavit of attorneys who are licensed to practice in the state May 16, 2011 page 28-29 2) In the DeBartolo case: Ct thought he was a liar b/c he put wrong school on the application and the number of parking tickets (lots of them even though they were paid), he also passed himself as an officer. Skip Problem #3 (due 4-6) due chapter 3. a) Trace the character investigative procedure that was followed. How were the facts determined? DeBartolos responses on his application and evidence introduced at the hearing (DeBartolos testimony, Classmates testimony) Who should bear the burden to show an applicants good (or lack of good) character? The applicant - practicing law is a privilege, not a right, and therefore, the burden should be the applicant b) Do you think that the decision in DeBartolos case was correct given the circumstances? Yes Rule 8.1 c) Should some conduct forever bar an applicant from becoming a lawyer? Yes (most felonies are) What about murder? Yes Child molestation? Yes Major securities or bank fraud? Yes Tax evasion? Yes Dozens of arrests at pro-abortion or anti-abortion demonstrations? No Drunken driving convictions? No d) If an applicant should be allowed to show rehabilitation, how long should the rehabilitation period be and who should decide? Can numerical standards be set? 5|Page

3) In the state in which you plan to practice law, will you have to become a member of: a) The state bar association? (If you must, what advantages and disadvantages do you see in compulsory bar membership?) YES the license to practice law is a membership in the state bar the privilege license for the state is $300 per year and is paid to the bar Bar commissioners govern the state bar and representatives are elected from each of the 44 circuits (minimum of one from each circuit; currently there are 58 representatives) the bar publishes the Alabama Lawyer once a month the bar has sections that deal with particular practice areas the bar carries out discipline through the grievance committee as the designee of the AL Supreme Court sponsors CLE program (12 hours CLEs required each year, 1 hour must be in ethics) carries out the IOLTA (Interest on Lawyers Trust Accounts) advisory opinions on ethics - AL has what is called an integrated bar: means that when you pass the test and are admitted to the Bar, you automatically become a member of the State Bar b) The city or county bar association in the area where you open your office? NO voluntary Brewer encourages lawyers to get involved good way to socialize with judges and senior lawyers c) The American Bar Association? NO voluntary governed by House of Delegates accrediting agency for law schools reviews all nominees for appointment to federal judgeships (3 ratings) publishes ABA Journal student memberships 4) Suppose that you have recently been admitted to practice before the highest court of State A. a) Client Arnold asks you to represent him in a lawsuit pending in the U.S. District Court for the Northern District of State A. Under what circumstances may you represent him? I must be admitted to the bar of that court. To do so I must be admitted in state A (which I am). Must fill out an application and pay application fee. Admission will typically be granted upon motion by an attorney who is already a member of that courts bar and who can affirm that I am a person of good moral character. b) Client Betty asks you to represent her in an appeal pending in the U.S. Court of Appeals for the circuit that covers State A. Under what circumstances may you represent her? I must be admitted to the bar of that court. To do so I must be admitted in the courts of any state. I am admitted in state A. Admission will typically be granted upon motion by an attorney who is already a member of that courts bar and who can affirm that I am a person of good moral character. 6|Page

c) Client Carlos asks you to represent him in an appeal that is pending in the U.S. Supreme Court. Under what circumstances may you represent him? I must have practice before the courts of a state for at least three years to be admitted to the bar of that court. d) Client Deborah, an economics professor who lives and teaches at a university in State A, asks you to represent her in a dispute over consulting fees for work she did for a business in nearby State M. You regularly represent and advise Deborah regarding her consulting work in State A. The dispute in State M appears headed for arbitration or other alternative dispute resolution proceedings in State M. Under what circumstances may you represent her? [See ABA Model Rule 5.5(a) prevents practicing in jurisdictions not licensed in.So we have Rule 5.5(c) and Comments 8 and 13]. Either Of: 5.5(c)(2)read comments 9,10,and 11. Associate with a lawyer admitted to practice there and comply with the law and who is active in the case Pro hac vice granted permission by jurisdiction.5.5(c)(2 very important) Alternative dispute resolution when dispute is arising in own jurisdiction or another if related to area of practice in own jurisdiction. Something incidental to work from your jurisdiction (e.g. deposition) as long as you comply with local rules Know R 8.5(a), (b) 5.5 Note 4 continuousgovernment lawyersome lawyers (i.e. immigration lawyers 5.5(d)(2) can practice anywhere). e) Client Edgar asks you to defend him in an automobile negligence case pending in State B, right across the river from your office in State A. Under what circumstances may you represent him? If I am admitted by reciprocity unlikely If I take the bar and become admitted in State B Pro hac vice admission each state has a procedure to be admitted in this way for this case only limits authority once admitted must be supported by a sponsor some states require you to associate with a local attorney (economic reasons and disciplinary accountability) [Note that you are subject to discipline in AL even if the conduct was in GA] Rule 8.5(a) if you practice in a state where you are not admitted to practice in, you are disciplined in that state and where you are admitted to practice. f) Your law partner, attorney Thomas, suggests opening a branch office of the firm in State B. Thomas is admitted to practice in both States A and B, but you and the other lawyers in the firm are admitted only in State A. See 5.5(b) note 4 says see 7.1 and 7.5(b) Under what circumstances may the firm open the branch office? [See ABA Model Rule 7.5(b)]. You may open an office in State B and use the same name as in State A, but identification of the lawyers in the office in State B must indicate the jurisdictional 7|Page

limitations on those not licensed to practice in State B (letterhead may have various asterisks identifying the various jurisdictional limitation of attorneys) 5) Lawyer Lawrence has come to you for legal advice. He has told you in confidence that he and a group of his friends formed a real estate investment venture. They entrusted him with a large sum of money to invest for them, but he diverted part of it for his own use. They have not yet discovered what he did, and he has asked you for legal guidance. a) If Lawrence was acting in the real estate transaction in his personal capacity, not as a lawyer, is he subject to discipline by the state bar for what he did? Yes Rule 8.4 (b) and (c) and Comment 2 An attorney is subject to discipline regardless of the capacity he was in when he performed the act. R 8.4(d) see note 3 8.4(a) Know (b) and (c) for exam and Bar. See Note 2 R 1.6 trumps everything (all other rules) except R 3.3 b) Do you have an ethical obligation to report him to the state bar? No Rule 8.3 (see note 2) creates a duty to report another attorneys professional misconduct but excludes information otherwise protected by Rule 1.6 (Confidentiality of Information). Lawrence is my client (Rule 1.18) so Rule 1.6 trumps. See Rule 8.3, Comment 3 gives discretion and allows a measure of judgment about what is serious (substantial and injurious) enough to report This duty to report applies broadly to partners, judge, and family members. o May make anonymous reports about judges o May allow the client to make the report if the client is aware of the conduct o May ask the lawyer guilty of the conduct to report himself to the bar o If you are unsure about what to report call Tony McLain o Judges have a similar provision under the judicial cannon to report lawyers who misbehave 6) In the Mountain case and the Holmay case, infra: a) How do you suppose the state bar first became aware of the misconduct? Unhappy clients reported it b) Trace the procedure followed in each case. How were the facts determined? Testimony Who decided what discipline to impose? The State Supreme Court 8|Page

c) Do you think the discipline imposed on lawyer Mountain was appropriate in the circumstances? Yes Rule 8.4 (Misconduct) he sold a baby, lied to clients and their attorney, had a conflict of interest, charged excessive fee d) Do you think the discipline impose on lawyer Holmay was appropriate in the circumstances? Yes Rule 8.4 (Misconduct) The Organization of the Bar Admission to the bar and lawyer discipline have traditionally been matters of state concern. Admission to Practice in the Courts of a State In most states, admission to practice law is gained by graduating from law school, passing the states bar examination, and demonstrating that you possess good moral character. Residency Requirements Since 1985 the Supreme Court has repeated struck down residency requirements. Supreme Court of New Hampshire v. Piper (U.S. 1985) N.H.s refusal to swear in a Vermont residence who has passed the states bar exam violated the constitutions Privileges and Immunities Clause. Frazier v. Heebe (U.S. 1987) The Court invoked its supervisory power in validate a residency requirement imposed by a U.S. District Court. Supreme Court of Virginia v. Friedman (U.S. 1988) The Court struck down a Virginia rule that let permanent VA residents licensed out-of-state waive into the VA bar, but required non-VA residents to take the state bar examination. Barnard v. Thorstenn (U.S. 1989) The Court struck down the Virgin Islands bars one-year residence requirement. Character Requirements May 17, 2011 All states require that an applicant for admission to the bar possess good moral character. Applicants are most likely to get into difficulty for dishonesty on the bar application, recent criminal conduct, and fraud or other financial misdeeds. In Re DeBartolo (Ill. 1986) p.33 o Facts: After DeBartolo passed the bar, the Committee on Character and Fitness refused to certify to the State Board of Law Examiners that DeBartolo possessed the good moral character and general fitness necessary for the practice of law. DeBartolo filed a petition for relief. DeBartolo had incurred 200 to 400 parking tickets He twice falsely represented himself as a police officer He provided incorrect information regarding high school attendance (incorrect name of school and years attended) He failed to list all address at which he had lived for the previous 10 years 9|Page

o Burden of proof is on the applicant to show good moral character. The practice of law is a privilege. o Result: The court denies the petition for relief and agrees that DeBartolo does not demonstrate the good moral character and general fitness necessary to qualify him for admission to the State bar. The court did provide that he could reapply at a later time and show rehabilitation. Admission to Practice in Other States and the Federal Courts An attorney who has been admitted in one state and who wants to represent a particular client in a court of another state may also petition that court to appear pro hac vice, i.e., for this turn only. o There is no right to appear pro hac vice. o A separate petition is required for each case. o Some jurisdictions require the attorney to enlist a local attorney as cocounsel to assure compliance with local procedure and provide accountability to he court Most states allow some limited practice activities by lawyers admitted in another state. Examples include in-house counsel, members of the military, for provision of pro bono services, and law faculty members. Most states have reciprocity arrangements that allow an attorney who has practiced in one state for a set number of years to gain full admission to practice in another state simply by filing a petition. An attorney who wants to practice in a federal court must be separately admitted to the bar of that court, because each federal court maintains its own separate bar. Typically, admission is granted upon motion by an attorney who is already a member of that courts bar and who can affirm the applicant is of good moral character. Admission to a federal district court, typically requires the applicant be admitted in the state in which the federal court sits. o Each federal district court has local rules available from West Publishing o Get admitted right away when you pass the bar File application Application fee ($50) Member of the bar must sponsor you o Must be renewed periodically by paying a fee Admission to a federal court of appeals requires the applicant be admitted in the courts of any state. o Federal Rules of Appellate Procedure 46(a) (?) o Application that contains oath o Fee ($20) o Must have sponsor will present you to the judge and you will take the oath verbally o Will receive a copy of internal procedures upon admission 10 | P a g e

Admission to the U.S. Supreme Court requires the applicant have practiced before the courts of a state for at least three years. o Supreme Court Rule 5 o Must have 2 sponsors who are members of the bar of the Supreme Court o Lifetime fee ($100) o May be admitted in a formal ceremony or by mail

Lawyer Associations Nationwide Organizations The American Bar Association State Bar Associations Many states require that all lawyers practicing in the state belong to the state bar association = mandatory or integrated bar Lathrop v. Donohue (U.S. 1961) Upheld Wisconsins compulsory bar membership does not violate either the right of free association or free speech Keller v. State Bar of California (U.S. 1990) an integrated bar must not use its members mandatory dues to fund activities of an ideological nature that are not germane to regulating the legal profession and improving the quality of legal services. Local Bar Associations Sources of Guidance on Legal Ethics State Rules, Statutes, and Rules of Court o Each state has a set of ethics rules o Some states have statutes that govern the conduct of lawyers o Most courts have local rules ABA Model Code of Professional Responsibility o In 1969 the ABA promulgated the ABA Model Code of Professional Responsibility as a model for the states ABA Model Rules of Professional Conduct o The ABA Model Rules were designed to replace the ABA Model Code ABA Code of Judicial Conduct o In 1972 the ABA promulgated the CJC as a model for the states to follow in adopting their own sets of rules for judges Advisory Opinions of Ethics Committees o The ABA and many state and local bar associations have ethics committees some publish their opinions they are not binding on any court or disciplinary body, but are often cited as authority Ethics Hot Lines o Some state and local bar associations provide quick ethics research service by telephone

11 | P a g e

Discipline Discipline refers to the penalties imposed by a disciplining agency on an attorney who has breached a rule or statute for which discipline can be imposed. o Violation of the disciplinary rules and some statutes o Crimes that reflects on honesty, trustworthiness, or fitness as an attorney o Conduct that involves dishonesty, fraud, deceit, or misrepresentation, or that prejudices the administration of justice, or for stating an ability to improperly influence a government agency or official, or for helping a judge violate the law or the rules of judicial ethics o Specific statutory violations (e.g. ambulance chasing, sex with client, advocating violent overthrow of the government) Three types of discipline: o Disbarment typically means permanent removal from the practice of law, but some states allow a subsequent petition for readmission o Suspension the attorney is prohibited from practicing law for the term of the suspension May included the requirement the attorney take and pass a legal ethics bar examination Suspension is sometimes stayed and the attorney is place on conditional probation o Reprimand (either public or private) Private reprimand unpublished, private communication in writing from the agency to the attorney Public reprimand published names the attorney and describes the improper conduct serves both educational and a warning function In Re Mountain (Kansas 1986) p.55 May 18, 2011 o Facts: Mountain represented two couples who were seeking to adopt the same child. He lied to one couple about the child and failed to represent them. He served as a procurer of a baby for adoption and collected an excessive fee ($17,000)[He had agreed to charge the first couple $500]. o Result: Disbarred He lied, conflict of interest and human trafficking In Re Holmay (Minnesota 1987) p.57 o Facts: Holmay forged (or procured a forgery of) his clients signature on documents (without clients permission), which he falsely notarized, submitted to a court, and served on the opposing party. o Result: Suspended for 30 days and ordered to pay $500 costs o The false notarization is the more serious charge. Notarization makes a document self authenticating. What probably happened above - consider below Hypo: Hypo You have a D.V. client and need a TRO on Friday afternoon. Client has left for Chattanooga without signing the affidavit. What can you do? Fax and have a 12 | P a g e

Chattanooga notary notarize the affidavit when the client signs. Talk to the judge and see if you, neighbors, or police can make an affidavit instead. May 18, 2011 CHAPTER 3 BEGINNING AND ENDING THE LAWYER-CLIENT RELATIONSHIP Pages 62-91 Rules ABA Model Rules: 1.16, 1.18, 3.1, and 6.1 -6.2 I. BEGINNING THE LAWYER-CLIENT RELATIONSHIP A. Duty a. b. c. B. Duty to take some kinds of cases General Rule: Lawyers Are Not Public Utilities Exceptions to the General Rule Pro Bono Public Service to reject some kinds of cases

II. ENDING THE LAWYER-CLIENT RELATIONSHIP A. GENERAL RULE: Duty to Follow through B. Mandatory withdrawal i. Fired ii. Client not in Good Faith iii. Violation of Disciplinary Rule iv. Lawyers Mental or Physical Incapacity C. Permissive Withdrawal D. Do I still Get Paid? III.Frivolous Claims Rules 1.16, 1.18, 3.1, 6.1, 6.2. 6.3 Discussion Problems 1) When Sheila was admitted to law practice she took an oath never to reject, for any consideration personal to [her]self, the cause of the defenseless or the oppressed. The American Nazi Party asked her to represent a member who was arrested for participating in an allegedly illegal street rally. Other lawyers in town have refused to get involved in the case. Sheila believes the citys refusal to issue a rally permit was a violation of the 1st Amendment, and she believes she could present an effective defense. She is repelled by the defendant and his political beliefs. She knows her reputation and her law practice will suffer if she takes this case. a) Does Shelia have an ethical obligation to take the case? She has no legal obligation to take the case. Lawyers are not public utilities. There are exceptions: There is a responsibility of pro bono work. Duty to take our fair share of unpopular or repugnant clients/causes. [Rule 6.2, Comment 1] If appointed by the court [Rule 6.2] she should not seek to avoid representing the client except for good cause [conflict of interest, attorney is a witness in the case, not competent, will 13 | P a g e

involve violation of a rule of professional conduct (client has informed you he will offer purgered testimony), unreasonable financial burden, cause is so repugnant that it will impair relationship or ability to represent]. She can probably claim financial burden or that cause is so repugnant. Excpetions- 3 situtations where we can ask the court to decline the appointment 1) Violation of Professional Conductconflict of interest under Rule 1.7 or 1.9 attorney is a witness under Rule 3.7 We know our client will commit purgery 3.3(a)(4) We are not competent to handle the casenever done a criminal case etc. under Rule 1.1 2) Appointment will result in an unreasonable burden to the lawyer Example- Real Estate Atty only does closing, and he couldnt close loans at his office if he was defending the case 3) Cause is so repugnant that it would likely b) Suppose the defendant were indigent, that the Party could not fund his defense, and that the Public Defender could not represent him because of a conflict of interest. If the court appointed Sheila to defend him, may she refuse? Rule 6.2 Not unless representing the client will result in an unreasonable financial burden on the lawyer or the client or the cause is so repugnant to the lawyer as to be likely to impair the client-lawyer relationship or the lawyers ability to represent the client. If the judge requires her to represent the client, Shelia has no choice. In Alabama we have 4 districts that have public defenders, so in all other places, lawyers are appointed to take these cases 2) You are a new partner in a small, general practice law firm. There is a proposal that would permit and encourage every lawyer in the firm to devote the equivalent of 100 billable hours per year to representing indigent clients. In order to adopt this proposal, the firm will have to cut overhead, cut pay, or raise fees to paying clients. How will you vote and why? Rule 6.1 I might suggest that instead of 100 hours we start with 50 hours. Perhaps this will not be so burdensome. Florida has no minimum requirement of pro bono hours, but requires attorneys to report the number of pro bono hours they work each year and encourages them to pay $350 to a legal aid organization of they do not choose to do pro bono work themselves Alabama appointed indigent defendant system representation of indigent criminal defendants is reimbursed by the state at $40 per hour for out of court work and $60 per hour for in court work some 14 | P a g e

counties contract out from criminal defense work some counties have public defender systems 3) Give five illustrations in which a lawyer would be subject to discipline for accepting or continuing employment in a matter. Rule 1.16(a) Duty to decline employment(mandatory) When the representation will result in violation of the rules of professional conduct or other law (frivolous claim, conflict of interest, client wants to commit crime or perjury, etc.) When the lawyers physical or mental condition materially impairs the lawyers ability to represent the client When the lawyer is discharged Rule 1.16(b) May voluntarily withdrawif withdraw can be accomplished without material adverse effect on the interests of the client or if The client persists in a course of action involving the lawyers services that the lawyer reasonably believes is criminal or fraudulent The client has used the lawyers services to perpetrate a crime or fraud A client insists upon pursuing an objective that the lawyer considers repugnant or imprudent The client fails substantially to fulfill an obligation to the lawyer regarding the lawyers services and has been given reasonable warning that the lawyer will withdraw unless the obligation is fulfilled [client will not pay] The representation will result in an unreasonable financial burden on the lawyer or has been rendered unreasonably difficult by the client Other good cause for withdrawal exists Rule 1.16(c) When ordered to do so by a tribunal, a lawyer shall continue representation notwithstanding good cause for terminating the representation [if in litigation, should obtain leave to withdraw as counsel the court may not allow withdrawal because of prejudice that would result to the client the court has the authority to supervise the litigation] Rule 1.18 When the client has interests materially adverse to those of a prospective client in the same or a substantially related matter if the lawyer received information from the prospective client that could be significantly harmful to that person in the matter (unless certain exceptions apply) Rule 3.1 When the attorney brings or defends a proceeding, or asserts or controverts an issue, and there is no basis in law or fact for doing so that is not frivolous. Rule 3.7 Attorney cannot represent client if the attorney is a necessary witness in the case 4) Judicial opinions sometimes state: A client has a right to discharge a lawyer at any time, with or without cause, subject to liability for payment for the lawyers services. Is there any situation in which the first part of that statement is not true? When the court will not allow it because it would disrupt a trial or cause a prejudicial delay 15 | P a g e

5) Simon agreed to represent Noreen in a personal injury suit in Florida. Their fee agreement provided for Simon to receive 20% of the net recovered by settlement, 30% of the net recovered after trial, or 40% of the note recovered after trial and appeal. The percentages were to be computed after deducting litigation expenses. In no event would the fee be less than $5,000 nor more than $10,000. After Simon spent about 100 hours on the case, Noreen fired him for no apparent reason. She repaid $1,000 for litigation expenses he had advanced on her behalf, but refused to pay him any fee. She hired another lawyer who promptly settled the suit for $13,000. The reasonable value of Simons 100 hours work is $6,000. What are Simons rights against Noreen? $5,000 fee $12,000 net recovery ($13,000-$1,000) / 20% of $12,000 = $2,400 / minimum contract fee = $5,000 / Quantum meruit = $6,000 if he had been fired for cause she would owe him nothing Three possible approaches: contract amount, quantum meruit, quantum meruit subject to minimum and maximum that could be recovered under the contract 6) Compare the Holmes case with the Kriegsman case. Do you disagree with the result in either case? NO Why was the attorney allowed to withdraw in Holmes, but not in Kriegsman? In Homes the defendants will have ample time to retain new counsel and the plaintiff will not be visibly prejudiced. In Kriegsman, if the attorneys withdraw the client will be harmed. BEGINNING THE LAWYER-CLIENT RELATIONSHIP Duty To Take Some Kinds of Cases General Rule: Lawyers are not public utilities o A public utility has a duty to serve anybody who wants service and can pay for it (Public utilities have a regulated monopoly) o A lawyer may reject work for any reason that suits her. Exceptions to the general rule o When you are admitted to law practice, you will take an oath, as required by state law WHO SHOULD PAY WHEN THE CLIENT CANNOT? Rule 6.1 Expresses the expectation that lawyers contribute a fixed number of hours per year to pro bono activities with 50 hours as the default number and that a substantial majority of the 50 hours be devoted to serving the poor either in person or through organizations. The Rule also calls upon lawyers to make voluntary financial contributions to organizations serving the poor. In criminal matters, public funds are generally available to provide modest compensation to private lawyers who represent indigent criminal defendants. Public funds are generally not available to compensate private lawyers who represent indigent persons in civil matters. 16 | P a g e

Bothwell v. Republic Tobacco Co. (U.S.D.C. of Neb. 1995) p.55 Motion submitted by plaintiffs appointed counsel seeking reconsideration and vacation of the judges order appointing her to represent plaintiff in this case. Facts: Bothwell submitted a request to proceed in forma pauperis, a civil complain, and a motion for appointment of counsel. Bothwell alleged he switched from smoking factory-manufactured cigarettes to his own rolled cigarettes, mistakenly believing they were safer because loose tobacco came without warning labels. The judge appointed Metcalf as plaintiffs counsel. Metcalf asked the judge to vacate the order of appointment. Result: grant the motion and vacate the order of appointment Rationale: o The judge held he did have the authority to make an appointment for a civil case for four reasons: When Client is indigent A federal district court does possess the inherent power to compel an unwilling attorney to accept a civil appointment. Why? To ensure a fair and just adjudicative process in individual cases To maintain the integrity and viability of the judiciary and of the entire civil justice system Attorneys are officers of the court as an officer should be responsive to the needs of the court Monopoly theory attorneys must provide legal services to indigents without compensation by virtue of the exclusive privilege they have been granted to practice law Attorneys have an ethical obligation to render services to the poor the flip side of this is the courts inherent authority to provide instruments to ensure fairness and justice, and to maintain the relevance of the court system in resolving civil disputes o This case was not appropriate for the exercise of that authority: A court must exercise its inherent powers with restraint and discretion the power should be exercised only where reasonably necessary for the administration of justice (could the plaintiff represent himself, could the plaintiff obtain counsel on his own, etc.) In this case, the plaintiffs failure to obtain private counsel was not the result of his indigency but rather a result of the marketability, or lack thereof of his claims. It is not reasonably necessary to the administration of justice for this court to compel Metcalf to represent the plaintiff. Note: in the Courts comment as to the lawyers claim that the courts inherent authority to enforce Her to represent Client would violate the Fifth and Thirteenth AmendmentWhile court did not address the issue it did commit that other courts have taken up the issue and have found No violation of the Constitution. 17 | P a g e

Alabama State Bar has a volunteer lawyers program. You may agree to handle a case pro bono or for a reduced fee. Attorneys sign up for particular types of cases. Clients are then referred to an attorney who will handle their case. OUTLOOK FOR THE FUTURE The courts are reluctant to find lawyers duty-bound to represent indigent clients without compensation The Supreme Court has never directly considered the constitutionality of mandatory pro bono duties. Ruskin v. Rodgers (App. Ct. of Ill. 1979) p.79 Legal Suit for specific performance on written agreement for the purchase of an apartment building and its conversion into condominiums. P prevailed and D appealed claiming numerous errors including the following (unfair trial because he was denied to substitute his attorney): Facts: The defendant requested a continuance two days before the previously set trial date. The trial court denied the motion. During the trial the defendant attempted to discharge his attorney. The trial court denied his motion for substitution of attorneys during the course of trial. Contentions: Defendant contends he was deprived of a fair trial. Defendant contends an individual has an absolute right to replace his attorney at any time with or without cause. Result: No error Rationale: o There was no manifest abuse of discretion o To allow substitution of attorneys at this point would have been extremely disruptive to the trial and would have resulted in a significant and prejudicial delay Rosenberg v. Levin (Fla. 1982) p.80 Facts: Levin hired Rosenberg and Pomerantz to perform legal services. The agreement provided for a $10,000 fixed fee, plus a contingent fee equal to 50% of al amounts recovered in excess of $600,000. Levin later discharged both without cause and subsequently settled the matter for a net recovery of $500,000. Rule: A lawyer discharged without cause is entitled to the reasonable value of his services on the basis of quantum meruit; but recovery is limited to the maximum fee set in the contract entered into for those services. o In contingency fee cases, the cause of action for quantum meruit arises only upon the successful occurrence of the contingency. Result: Here quantum meruit would be $55,000, but the court said he cannot profit by being fired. He is limited to $10,000 fee. Holmes v. Y.J.A. Realty Corp. (N.Y. 1987) Pg 84 Appeal from denial of attorneys motion to be relieved as counsel for the defendants. 18 | P a g e

Facts: A written retainer agreement provided that legal fees would be billed periodically at the rate of $125 per hour for law office activity and $400 per day for each court appearance. The balance due from these clients if $2,275.30. Defendants have refused to make payment. Result: The motion to withdraw is granted. Rationale: o Once representation of a client in litigation has commenced, counsels right to withdraw is not absolute. o An attorneys withdrawal is permissible where a client renders it unreasonably difficult for the lawyer to carry out his employment effectively. o An attorneys withdrawal is permissible where a client deliberately disregards an agreement or obligation to the lawyer as to expenses and fees o Defendants will have ample time to retain new counsel. Plaintiff will not be visibly prejudiced.

Kriegsman v. Kriegsman (Superior Court of N.J. 1977) p.85 Appeal from an order denying their application to be relieved as attorneys for plaintiff Facts: Rose firm alleged it had spent 110 hours on this case, billed at $7,354.50 and had incurred disbursements of approximately $242. Plaintiff was now on welfare and unable to pay. The Rose firm alleged they were entitled to be relieved from further representation. Plaintiff opposed. This case had been long and complex and was about to be ready for trial. Result: Affirmed attorneys are not relieved Rationale: o When a firm accepts a retainer to conduct a legal proceeding, it impliedly agrees to prosecute the matter to a conclusion. The firm is not at liberty to abandon the case without justifiable or reasonable cause, or the consent of its client. o With trial imminent, it would be extremely difficult for plaintiff to obtain other representation, and therefore she clearly would be prejudiced by the Rose firms withdrawal. o The lawyer should not throw up the unfinished task to the detriment of his client except for reasons of honor or self-respect. FRIVOLOUS CLAIMS ABA Model Rule 3.1 prohibits an attorney from taking a frivolous legal position one, under the facts, that has no basis in existing law and that cannot be supported by a good faith argument for extending, modifying, or reversing the existing law. ABA Model Rule 1.16 an attorney must refuse employment (or must withdraw from employment) if the employment would require the attorney to violate a disciplinary rule or other law Suit for malicious prosecution elements: o The initiation or continuation of the underlying action 19 | P a g e

o Lack of probable cause o Malice o Favorable termination of the underlying action Sanctions o FRCP Rule 11 o 28 U.S.C.A. 1927 o Inherent power doctrine federal courts have the inherent power to sanction bad faith conduct by lawyers and parties whether the conduct at issue is covered by one of the other sanctioning provisions or not. o Similar sanction provisions are available under state law rules or statutes.

***Gaines, Gaines & Gaines, P.C. v. Hare, Wynn, Newell & Newton (Ct. of Civ. App. Ala. 1989) Facts: Did not cover o Thomas Benton Barnett (minor) died in an automobile accident that involved a driver of Floyd & Beasley. In total there were 4 people killed or injured. o Thomass father (Mr. Barnett) spoke with the father of another victim who had employed Hare, Wynn and recommended them to Mr. Barnett. o After a determination of negligence, Mr. Barnett allowed his son, Robert Jr. (who worked for Gaines, Gaines, & Barnett), to attempt settlement negotiations with the insurer of Floyd & Beasley. There was no fee arrangement. o When a settlement could not be reached, Robert Jr. contacted Hare, Wynn to prosecute the wrongful death action and arranged for a 25% contingency fee split equally between the two firms since Robert Jr. would continue an active role working on the case. o Two months later, Robert Jr. left his firm, moved, and no longer participated in the case. o Mr. Barnett sent a letter confirming the Gaines firm discharge and requested an invoice for any expenses incurred by the firm. o The case settled for $1,500,000. o Gaines firm filed motion to enforce fee agreement and establish an attorneys lien Procedural History: o The trial court held that any contingent fee agreement was terminated when Mr. Barnett terminated the Gaines firm and, further, than any agreement between the firms was contingent on the active participation of the Gaines firm, which was not fulfilled. o The Circuit Court held the firm was limited to a quantum meruit fee. The firm appealed to the Supreme Court. o The Supreme Court transferred the case to the Court of Civil Appeals. Result: Affirmed (1) the trial courts refusal to enforce a fee division agreement conditioned on active participation of firm was not an abuse of discretion, and (2) evidence supported amount of quantum meruit award 20 | P a g e

Precedent: Law firm which had been discharged was entitled to fee recovery based on quantum meruit, rather than the one-half of contingent fee it arranged to split with successor firm prior to discharge; any fee division agreement was conditioned on active participation by predecessor firm, and predecessor firms work was insignificant when compared with that of the successor. Rationale: o Effect of the discharge of the Gaines firm on the original contingent fee arrangement: Employment of an attorney by a client is revocable by the client with or without cause and ordinarily does not constitute a breach of contract Mr. Barnetts discharge of the Gaines firm left him free to contract employment with any other attorney and terminated the Gaines firms participation in a contingent fee. o The award to the Gaines firm of attorneys fee based on quantum meruit Part performance of a contract, prior to being discharged, entitles one to recover on quantum meruit for those services rendered. The involvement of the Gaines firm was due solely to the fact that the brother of the deceased was a partner. The brother did maintain an active role in the case. The Gaines firm did not continue to maintain any activity on the case after the brother left the firm. Credible evidence supports the trial courts finding that any fee division agreement was conditioned on active participation by the Gaines firm. when the firm was discharged, active participation became an impossibility and limited the Gaines firms recover to the services it had preformed There was no case referral by the Gaines firm to Hare, Wynn Awarded attorneys fees of $7,500 based on quantum meruit.

Gamble v. Corley, Moncus & Ward, P.C. (Ala. 1998) Did not cover Former associate attorneys resignation from law firm prevented law firm from satisfying condition precedent of referral agreement between former associate and new attorney requiring law firms continued participation in contingent fee case through former associate. The law firm was entitled to be compensated only on quantum meruit basis for its participation in case up until former associate resigned; new attorney would not have agreed to 55%/45% split of contingent fee without former associates promise that he would participate in case and share workload. CHAPTER 4 ADVERTISING AND SOLICITATION Discussion Problems ABA Model Rules 7.1 through 7.6. 1) In which of the following ways may you seek to build your clientele? 21 | P a g e

7.3(a) NO Real Time Contact 7.3(b) NO Contact

a) May you join a social club for the sole purpose of meeting new people and luring them as clients? YES. This is a traditionally accepted method of meeting people and becoming an integral part of the community. However, cant directly solicit fee paying business. However, if the person at the club asks if they can hire you, then youre ok. Rule 7.3. b) May you call on other lawyers at their offices and let them know that you are willing to take on work that they are too busy to handle? YES. This is permitted solicitation. This is networking. It is good to get to know other attorneys. Rule 7.3(a)(1) c) May you volunteer to give a seminar on estate planning for the local chapter of Young Businesswomen of America, hoping to get legal business from some of those who attend? YES. This is a good way to volunteer and get known. Bar CLE seminars are another good way. But, make sure you are competent, dont use it as a blatant advertising purpose, and do not use it as an opportunity to solicit in person. Rule 7.3 d) May you list your name with the local court as a person who is willing to take court-appointed cases? YES. This is a good way to get some experience and make some income. It is also good to get to know the other court officials. People often come to them looking for an attorney. (except as barred by Rule 7.6) May you contribute to the re-election campaign fund of a local judge, hoping to secure good court-appointments from the judge? NO Rule 7.6 pay to play you may make a political contribution for a legitimate reason, but you cannot do so if the intent is to have the judge reward or repay e) May you list your name with the lawyer referral service run by the local bar association? YES Rule 7.2(b)(2) How does such a lawyer referral service operate? Lawyers may register and may select the particular areas of law in which they wish to receive referrals. Referral service gets a fee. The referral is given several attorneys names and may choose to contact the attorneys. The attorney is given the name of the prospective client, but is not to solicit them. Martindale Hubble usually does not rate a new attorney for about five years then ratings can go from CV to BV to AV Collection directories agree to send collection account cases in your county to you you cannot split your fee with them your contact is usually with the collection agency and not directly with the client f) May you place advertisements for your services? YES Rule 7.2(a) In what media? Written, recorded or electronic communication, including public media Rule 7.2(a) What restrictions are there on the content of your advertising? It may not be false or misleading communication Rule 7.1 Technically True but Misleading admitted to practice before the US Supreme Court (misleading because it makes an attorney look to be overstating qualifications)

22 | P a g e

Last five cases have gotten $5M (makes the prospective client think its typical) o Can do this if you have a disclaimer Cant compare your services to other legal services (cant be factually substantiated) State cannot regulate communication that is not false or misleading unless: 1. Substantial Govt Interest Privacy and tranquility of PI victim Reputation of lawyers 2. Narrowly Tailored 3. Regulation has to directly and materially affect the governments substantial interest g) May you publish a brochure that describes your law practice, states the kinds of matters you handle, and provides a schedule of the fees you charge for a variety of routine legal services? YES Rule 7.4; can say you are a specialist in an area as long as it is not false or misleading (take a lot of cases in the matter, etc); certified specialist (certified by SBA; ABA) otherwise cant say youre certified May you put the same information on a World Wide Web site? YES May you use www.winbig.com as your site address? NO this would probably be a misleading communication Rule 7.1 (gives an unjustified expectation to potential clients) Super Lawyers? Might be false or misleading because it is a paid deal h) No 2) Lawyer Lovette would like to represent afflicted plastics workers who wish to bring legal proceedings against their employers and the manufactures of DNXP. a) May she put an ad in the local newspaper, informing plastics workers of their legal rights respecting exposure to DNXP and inviting interested Shadowland persons to contact her? YES Zauderer Zauderer could not be Cases disciplined simply for placing an ad that concerned a specific legal problem and that was designed to lure a narrow group of potential clients. Different from Bates because it wasnt general; it was specific to a certain segment of the population through a generally circulated paper. OK as long as truthful and not misleading. Didnt rise to the level of solicitation. b) May Lovette send an informative letter via the postal service to each plastics worker, inviting the worker to contact her for further information? YES Shapero v. Kentucky Bar Assn, (U.S. 1988) solicitation letters were more analogous to targeted newspaper ads than to in-person solicitation a state cannot ban solicitation letters outright, but it can impose reasonable regulations on their use e.g. she must tell them where she got their name and information and the envelope must say 23 | P a g e

advertisement. 7.3(c) UNLESS recipient is another lawyer, family friend, personal friend, or someone with whom youve Recorded v. Real had a professional relationship. Time - easier to Is your answer the same if Lovette uses email rather than the postal police recorded service? YES because there is a c) May Lovette hire a team of telephone callers who will use the track; harder in business directory to phone each plastics worker, give a brief real time b/c no description of the DNXP problem, and invite the worker to contact record Lovette for more information? NO Rule 7.3 and Rule 8.4(a) [cannot induce someone else to violate the Rules of Professional Conduct] d) May Lovette stand on the public sidewalk outside the gates of one of the plastics factories at quitting time and pass out handbills that state her willingness to represent workers in DNXP cases? NO Rule 7.3(a) Real-Time Communication (equivalent to Ohralik); however, she could hire someone to pass out pamphlets saying the same thing May she initiate conversations with workers on that subject? NO Rule 7.3(a) May she initiate such communications in a real-time Internet chat room? NO Rule 7.3(a) o You can use recorded phone message. Not real-time contact 3) On your way down the courthouse hall after a hearing, you saw a tiredlooking woman holding a crying infant. She was obviously confused and needed help. When you spoke to her, she handed you a paper and asked in halting English where she was supposed to go. The paper was a summons to appear that morning in an unlawful detainer action filed by her landlord. When you responded to her in her native language, her face broke into a wide smile. You briefly explained to her the nature of an unlawful detainer hearing, and you asked if she had a lawyer. When she said no, you offered to represent her at the hearing for a modest fee. Was your offer proper? NO this is in person solicitation Rule 7.3 if motive is pecuniary gain then no bueno Would it be proper if you had offered to represent her for free? YES Rule 7.3 there is no motive for pecuniary gain Biggest criticism of 7.3(a) is that there is no distinction between good solicitation and bad solicitation HISTORICAL SUMMARY Advertising In the 1800s, lawyers in the U.S. sometimes advertised their services in newspapers. When the ABA adopted the original Canons of Professional Ethics in 1908, solicitation and advertisement were condemned. Lawyers were to be passive receivers of legal business, not active seekers of it. 24 | P a g e

Between 1910 and 1975 the bar and courts created a remarkable body of law and lore. Rule 7.1 Communications may not be false or misleading o Communication is false or misleading if it contains a material misrepresentation of fact or law or omits a fact necessary to make the statement considered as a whole not materially misleading Rule 7.2 no real limits on types of media that may be used AL is stricter must send a copy of any advertisement within 3 days to state bar, must keep a copy for 6 years, the advertisement must include a name and address of the attorney responsible, must include a disclaimer regarding the quality of legal services

Solicitation Solicitation client-getting activity that involves personal contact (face-to-face contact, live telephone contact, or real-time electronic contact) which is initiated by a lawyer (or the lawyers agent) and a specific potential client Rule 7.3(a) prohibits solicitation by in person or live television or real time electronic contact qualified: when motive is for pecuniary gain and unless the person contacted is a lawyer, family, or someone with whom there is a prior professional relationship o Advertising general communication with the public at large o Rule 7.2 you can use any type of media for advertising Solicitation has traditionally been punished more harshly than advertising The Forces of Change Antitrust and the First Amendment The ABA Model Code of Professional Responsibility (1969) banned advertising and solicitation. However, it simultaneously spoke of a lawyers ethical obligation to help lay people recognize legal problems and to assure that legal service was available to all who needed it. Tension between these two positions Anti-trust experts began to ponder the anti-competitive effects of the lawyer advertising ban. The U.S. Justice Department sued the American Bar Association as a conspiracy of trade in 1976. The Bates Case Bates v. State Bar of Arizona, (U.S. 1977) 86 Facts: Bates and OSteen opened a legal clinic and ran a newspaper ad. The State Bar of Arizona tried to discipline them. They appealed to the U.S. Supreme Court, claiming violations of the Sherman Act and First Amendment free speech clause (as applied to the states through the Fourteenth Amendment). Issue: Whether the State may prevent the publication in a newspaper of appellants truthful advertisement concerning the availability and terms of routine legal services. NO Such information may not be restrained. 25 | P a g e

Result: o The advertising ban was immune from attack under the Sherman Antitrust Act because the ban had been promulgated by an arm of the state government. o The 1st Amendment commercial speech doctrine protects attorney advertising that is truthful and not misleading.

HISTORICAL SUMMARY, CONTINUED The Ohralik and Primus Cases Solicitation, p.94 Ohralik v. Ohio State Bar Association, (U.S. 1978) approved indefinite suspension of an ambulance chaser o A state may adopt prophylactic rules that forbid in-person solicitation of fee-generating legal business under circumstances that are likely to produce fraud, undue influence, or similar evils. o Pecuniary Gain for Ohralik o Commercial speech not as protected as political speech o The reason for prohibiting in-person solicitation is to avoid overreaching, coercion, duress In re Primus, (U.S. 1978) attorney wrote to prospective client to convey an offer of legal help because the ACLU had offered to supply free legal counsel the First Amendment protects collective activity undertaken to gain meaningful access to courts and states may not regulate such political speech without showing actual abuse the key here is there was no motive for pecuniary gain The Adoption of the ABA Model Rules A few weeks after the Bates case was decided, the ABA amended the ABA Code to loosen the advertising ban a little, but not much. In 1983 the ABA Model Rules contained liberal provisions on advertising and solicitation to comply with In re R.M.J. o In re R.M.J. under the commercial speech doctrine a state may flatly prohibit lawyer advertising that is false or misleading, and a state may regulate advertising that is not misleading if the state can demonstrate that the regulation directly serves a substantial state interest, and that the regulation is no more extensive than is necessary to serve that interest The advertising and solicitation provisions of the ABA Model Rules have been amended several times to keep up with the Supreme Courts decisions The Peel Case Claims of Specialization The organized bar has been slow to recognize specialization The original ABA Model Rules allowed a lawyer to tell the public she practiced in a certain field, or restricts her practice to a certain field, but prohibited a lawyer from stating or implying that she is a specialist in a field, subject to three exceptions o Patent attorney 26 | P a g e

o Admiralty practice o Certified Specialist in a field of law if she had been certified by the bar of her state Peel v. Attorney Reg. & Disciplinary Comn, (U.S. 1990) a lawyer who is certified by a private organization may call himself a certified specialist, provided he identifies the organization that certified him and takes related steps to avoid misleading the public Comments of ABA Model Rule 7.4 an attorney may state she is a specialist, that she practices a specialty, or that she specializes in particular areas may state she is certified as a specialist in a particular field only when the certifying organization (which must be clearly identified in the communication) has been accredited by the ABA or has been approved by an appropriate state authority

The Zauderer, Shapero, and Went for It Cases The Shadowland Between Advertising and Solicitation, p.97 Zauderer v. Office of Disciplinary Counsel of the Supreme Court of Ohio, (U.S. 1985) Zauderer could not be disciplined simply for placing an ad that concerned a specific legal problem and that was designed to lure a narrow group of potential clients. First Amendment protection extended to this commercial speech that is not false or misleading. Can target particular groups! Shapero v. Kentucky Bar Assn, (U.S. 1988) solicitation letters were more analogous to targeted newspaper ads than to in-person solicitation a state cannot ban solicitation letters outright, but it can impose reasonable regulations on their use Florida Bar v. Went for It, Inc., (U.S. 1995) upheld a Florida rule that prohibits lawyers from mailing solicitation letters to victims and their families for 30 days following an accident There has been no approval of in person solicitation. Florida Bar v. Went for It, Inc., (U.S. 1995) 99 Floridas response to Shapero: Florida did a study regarding attorney advertising and prohibited personal injury lawyers from sending targeted direct-mail solicitations to victims or their relatives for 30 days following an accident or disaster The Court upheld this prohibition because it was narrowly drawn Norris v. Alabama State Bar, (Ala. 1991) Sending flowers to a funeral home along with a note to decedents family offering assistance and unapproved brochure explaining law firms services, constitutes improper solicitation of prospective clients, warranting two-year suspension Brewer believes if this had gone to the U.S. Supreme Court it would have been reversed because of Shapero 27 | P a g e

CHAPTER 5 ATTORNEY FEES AND FIDUCIARY DUTIES Discussion Problems 1. After graduating from law school threes years ago, Delores became a solo practitioner in a medium-sized city. She has a general civil and criminal practice, and she puts in about 60 hours a week, which yields about 40 billable hours. She takes two weeks of vacation in the summer and another two weeks spread throughout the year. She and some other solo practitioners share a nicely furnished office under a lease-service agreement. The leasing company provides them a law library, receptionist, secretary, delivery and photocopy services. Dolores overhead and lease-service fee totals $7,000 per month. Delores is approached by a painter who wants her to represent him in a K dispute worth $6,000. She does not have expertise in this area of practice and will need to spend a couple of hours researching that a lawyer experienced in this are would not need. She estimates that the case will require about 10 billable hours (in addition to two research hours). The average lawyer in the community would charge $1500 and the painter does not want to pay by the hour but wants a flat rate fee. Rule 1.5 a fee must not be unreasonable Factors to consider: The time and labor required, the novelty and difficulty of questions involved, and the skill requisite to perform the legal service properly The likelihood, if apparent to the client, that the acceptance of the particular employment will preclude other employment of the lawyer The fee customarily charged in the locality for similar legal services The amount involved and the results obtained The time limitations imposed by the client or by the circumstances The nature and length of the professional relationship with the client The experience, reputation, and ability of the lawyer or lawyers performing the services Whether the fee is fixed or contingent a. What do you think would be a fair fee for Delores to charge Leonard? 40 hours X 48 weeks = 1920 hours $84,000 overhead divided by 1920 hours = $43.75 $80,000 income divided by 1920 hours = $41.66 $43.75 + $41.66 = $85 per hour hourly fee About $950 is what you come up with given the above numbers. How do you arrive at that figure? 28 | P a g e

b. Suppose Delores quotes Leonard a flat fee of $1750 and they make an oral agreement to that affect. Would this subject Delores to discipline? Shall (MUST) be communicated. Also MUST communicate in any changes in fee or expenses to the client. Preferably should be in writing (but not required) Rule 1.5(b) Brewer good way to confirm the fee arrangement is through a client letter thanking the client for engaging you in the matter c. Suppose that Leonard want a contingent fee rather that a flat fee. Delores agrees to take the case for 35% of whatever amount she recovers for Leonard. Under what, if any circumstances would that be proper? Rule 1.5(c) Must be in writing and signed by the client, state the method of by which the fee is to be determined, must notify the client of expenses the client will be responsible for. Upon conclusion of the matter, the lawyer must provide the client with a written statement stating the outcome of the matter and if there is recovery showing remittance to the client and the method of its determination (but the remittance does NOT have to be signed by the client). Contingent fee would be improper if the case is a sure winner If payment is contingent on getting someone a divorce or the amount of support/alimony Not proper However, lawyer can have a contingent fee agreement if the lawyer is helping a person collect on alimony or support 1.5(a) also applies to contingent fee agreements (those factors apply) Look at contingency under the circumstances and evaluate it in light of the 1.5(a) factors Reasonable AT THE TIME that the contingency fee was signed, NOT hindsight d. Suppose that Leonard paints portraits rather than houses, and that the amount in controversy is $60,000 rather than $6,000. How, if at all, would that change your estimate of a fair fee? This case may be more difficult to prepare, and thus may require more time. You may need expert advice or expert testimony. These expenses should be taken into account. Another factor that may be considered is the amount involved. Brewer thinks about $5,000 - $6,000 would be appropriate. This would be a good case in which to suggest a contingent fee. 2. The law firm of P,S,R-M & T limits its practice to business law and much of the firms work comes from two big clientsUniversal Telecom (worldwide supplier of modern communications equipment and services) and ShopRite, Inc. (a nationwide chain of department stores.) Tara Gunn was fresh out of law school when she joined the firm as an associate. Due to the low cost of living in the region, Taras starting salary was equivalent of new associates salaries in NY and other major cities. 29 | P a g e

During her first year at the firm, Taras picked up the following pieces of information about the firms billing and promotion policies. React to each piece of information: a. Billing is handled in the firm by the partner in charge of each matter who totals the number of hours worked and multiplies by the billing rate for that lawyer. Adjustments are made for difficulty of the work, skill of the lawyer working, the result achieved and the amount at stake. Adjustments can go either up or down and the customer is aware of that. When a customer has a concern over billing, the lawyer will review the bill with them and work it out. This is standard and appropriate. b. Another associate at the firm only logged 10 billable hours on a project that took him over 30 hours because the project turned out to be simple once he found the right cases and if he had taken IP classes in law school he would have known that. He didnt want the boss to think he was a dolt. This is inappropriate. He should tell his boss he spent 30 hours. The hourly rate should take into account that the attorney is inexperienced. The boss can decide how many hours should be charged for in this case. c. While interviewing for a her job at the firm, she was told that the firm wants the lawyers to have time for their families and other activities and that they normally expect their associates to bill 1800 hours per year. Additionally, she was told that associates receive raises and bonuses commensurate with their performance and growth as determined by the Budget Committee. Once she came to work in the firm she was told that associates who log 1800 billable hours receive only a token bonus and salary increase. The big money goes to those who log between 2000 and 2400 hours and that those logging only 1800 seldom make partner. This is something you should look at when making a decision about where to practice law. d. Allison (more experienced associate) told Tara that she should always bill 8 hours in a day and 40 hours in a week at a minimum. She said that one approach is to make five two-minute phone calls in order to have 30 minutes of billable time (the firm charges a minimum of 6 minutes per phone call) and to keep track of think and worry time on the case while you are occupied with other activities but while your mind is still on a problem for a client. She suggests billing that as review and analysis or Preparation for deposition of Phone calls are ok; however if all the phone calls are for the same client, then probably only bill .2. Should not charge for worry time. e. Another associate advised Tara to keep track of her billing as she changes each task during the day. He suggests that saving it to the end of the day is more complicated and that you will forget the small details of what you have done during the day. Second, he suggests that Tara systematize her work based on the patterns that she sees in the projects that she does. He has developed forms and checklists for the five basic problems that he deals with and is able to churn out the work in a third of the time that he used to spend. However, he still bills it out at the rate of speed he used to produce it, so now he can bill 2300 hours 30 | P a g e

a year. He is happy, the boss is happy and the client is happy. You should be systematic and log your time before you forget. His charging scheme is inappropriate and dishonest. He has not spent the time he is billing. The benefit should go to the client; you cant bill the time it used to take you. Also cannot charge a higher flat fee reasonable under 1.5(a) factors. 3. Dringle bought a sailboat and a motorboat from Seaboard Marine Supply Company on credit, and he failed to make timely payments on either boat. Seaboard hired attorney Welch to collect the two debts. Welch sent a series of appropriate demand letters to Dringle. Finally, Dringle left a check, a message, and a set of keys at Welchs office. The message said: Her is a check for $7500 I owe to Seaboard on the sailboat. I have no way to pay for the motorboat, so here are the keys to it; you will find it tied to the dock at the City Marina. At the time, Welch was in trial and working 16 hours a day, so she put the message, the check and the keys in an envelop labeled Seaboard Marine, and put the envelope in her office safe. Her trial ended two weeks later, and that very day she deposited the check in her client trust account and notified Seaboard about the keys and the location of the Motorboat. Did she handle the matter properly? NO Rule 1.15 she should have notified the client promptly and delivered the property and funds to the client promptly (d) could have the secretary handle the deposit and check to make sure the motorboat is really there and call the client(a) she still has a responsibility to this client regardless of whether she is in court Is she subject to discipline? YES She violated 1.15 there does not have to be damages to discipline, but it is unlikely there would be a complaint if there was no loss Is she subject to malpractice if the check bounces and the boat is gone after two weeks? YES she violated the standard of care 1.15(b) can add your own money into the trust account only to the amount necessary to pay service charges on the account. An advance of fees or expenses all goes into the client trust account and then you can pay yourself out of that in pieces

4. Attorney Arner agreed to represent client Corman in a suit against defendant Drews. Arners written agreement with Corman provided that Arner would receive $75 per hour for his work, and that the fee may be deducted from the proceeds of the litigation before payment thereof to Corman. Corman won a judgment for $50,000 and Drews sent Arner a check in that amount made payable to Corman. Arner consulted his time records and concluded that he had spent 153.3 hours on the case, for a total of $11,500. Arner endorsed the check with Cormans name and presented it at the bank where he maintained his client trust account. He had $38,500 deposited in the client trust account, and he took the other $11,500 in the form of a bank cashiers check, made payable to him personally. That same day, he wrote to Corman as follows: 31 | P a g e

Drews check came in today. I have expended 153.3 hours on the matter, for a total of $11,500. Accordingly, I am holding $38,500 for your account, and I will send you a check in that sum promptly upon receiving word from you that the fee computation is correct and in accordance with our agreement. Did Arner handle the matter properly? NO he should not have endorsed the check without authority; would need a power of attorney to do that otherwise its fraud Was it okay to take his fee out before notifying the client? YES He promptly notified the client with the letter and gave an accounting, even though an accounting was not requested. He should have sent the client an itemized bill FIRST. Then, if the client okays it, then ARNER could take the fee pursuant to the bill/invoice. If the client disputes the fee taken, the attorney should replace the disputed amount to be kept separate until the dispute is resolved. In reality, usually checks are made to the attorney or the attorney and client together. If you get a check made to the client, the client should endorse the check. You should contact the client and have the client come in and endorse the check and have a letter ready for the client that gives an accounting and ask the client to sign agreeing to the fee. Then give the client the check to endorse and deposit it for him. This gives you consent to take your fee and allows you to make the deposit. Four types of fees: Flat Hourly Contingent Mixture value based billing, flat fee + contingency, etc.

Brewer Be sure when you make a fee agreement with a client that you get it in writing and provide for circumstances that are likely to arise (e.g. In the even that a counterclaim is filed). Two basic types of retainers: Non-refundable retainer an engagement fee (buying a commitment); sometimes may cover preliminary work this money is earned at the time it is received and does not go into client trust account Refundable retainer deposit attorney agrees to bill against the amount at an hourly rate any amount that is not used is refunded is placed in client trust account and is drawn out as it is earned Robert L. Wheeler, Inc. v. Scott, (Okla. 1989) 113 Facts: Wheeler represented Scott in a collection and lien foreclosure file against Scott. Wheeler billed Scott for 1295.9 hours, most of which were the work of a first year associate who had just recently passed the bar. Issue: Whether, after summary judgment was entered against Scott (client) in a mortgage foreclosure proceeding, and after the trial court subsequently reduced the fee charged by Scotts attorney from $140,116.87 to $125,723.00, the fee was still excessive. YES 32 | P a g e

Twelve Factors in determining the reasonableness of attorney fees o Time and labor required o Novelty or difficulty of issues o The skill requisite to perform o Loss of opportunity for other employment o The customary fee o Whether the fee is fixed or contingent o Time limitations imposed by the client or circumstances o The amount involved and the results obtained o Experience, reputation, and ability of the attorney o The undesirability of the case o Casual or regular employment o Awards in similar cases Result: R/R The fee was excessive. In this case the trial court gave too much weight to the time spent on the case and failed to consider adequately the other applicable standards. Much of the time expended was unnecessary by any reasonable standard.

ABA Formal Opinion 93-379, p.117 Several practices that are the subject of frequent inquiry: o Billing more than one client for the same hours spent When the basis for billing the client has been agreed to, the economies associated with the result must inure to the benefit of the client. o Billing for expenses and disbursements Reasonableness standard should apply to these charges o What are the services and costs for which a lawyer may legitimately charge General overhead in the absence of disclosure to the client in advance to the contrary, the client should expect that the cost of maintaining a library, securing malpractice insurance, renting office space, purchasing utilities, and the like are subsumed within the charges the lawyer is making for professional services Disbursements Should generally be passed on to the client, but in the absence of disclosure to the contrary, it would be improper to assess a surcharge over and above the amount actually incurred unless the lawyer herself incurred additional expenses beyond the actual cost of the disbursement item In-house provision of services photocopying, computer research, on-site meals, deliveries, other similar items may pass on reasonable charges for these services At the outset of the representation the lawyer should make disclosure of the basis for the fee and any other charges to the client. A lawyer has an obligation to provide competent representation to a client. Competent representation requires the legal knowledge, skill, thoroughness and preparation necessary for the representation. 33 | P a g e

Notes: Billing costs at an inflated rate is inappropriate unless the client has previously agreed to those charges. Generally, you should not charge the same time to more than one client. When time is spent on more than one client it is fair to divide the time between the clients. There should not be double dipping. You can charge two clients for the same time when one of the clients is being charged at a reduced rate for travel time. Brewer recommends Professor Ross book on billable hours. The Billable Hours Treadmill Flaws in the billable hours model: o The billable hours model doesnt encourage the lawyer to prepare a project plan or case plan at the outset o The billable hours model doesnt enable the client to predict how much the legal services will cost o The billable hours model penalizes the efficient, productive lawyer and rewards the slow plodder who racks up many billable hours o The billable hours model may discourage communication between lawyer and client Alternative methods of charging clients and paying lawyers: o Fixed or flat fees o Contingent fees o Hybrid billing methods Flat fee plus an hourly rate for work in excess of a specified maximum Hourly fee plus a contingent fee Questions and Answers About Contingent Fees Contingent fees may be used in many types of cases, not just personal injury In non-litigation matters, the contingency is the amount saved, rather than the amount gained ABA Model Rule 1.5 o A lawyer must not use a contingent fee when defending someone in a criminal case o A lawyer must not use a contingent fee in a domestic relations case if the contingency is getting a divorce, or the amount of alimony, support, or a property settlement (may use a contingent fee for legal work done to collect unpaid money due under an alimony or support decree or a property settlement) [public policy reasons the state has an interest in preserving marriage] A contingent fee must be reasonable: o A contingent fee can be higher than an hourly fee or a fixed fee because the lawyer bears the risk of not being paid at all if the matter is lost 34 | P a g e

o Reasonableness is measured at the time of making the contingent fee arrangement, not at the time the fee becomes due. o The sure winner situation is the one in which a court is most likely to conclude that a contingency fee is unreasonable Contingent fees can be either static or sliding

Loans to Clients Rule 1.8(e) A lawyer should not provide financial assistance to a client in connection with pending or contemplated litigation. There are two exceptions A lawyer can advance court costs and expenses of litigation (repayment contingent upon outcome) A lawyer representing an indigent client can pay court costs and expenses on behalf of the client AL does not have this rule, but does have a rule that you cannot promise help in order to induce business. You can help the client once you have taken the case. Client Trust Accounts ABA Model Rule 1.15 requires attorneys to keep clients money and property separate from their own, to maintain adequate records, to notify clients promptly when money or property is received on their behalf, and to deliver promptly any money or property that belongs to clients Must keep records for 5 years according to Model Rules (6 years in Alabama) Can put seed money in trust account minimum to open the account Must keep money is dispute separate until the dispute is resolved IOLTA Interest on Lawyer Trust Accounts State bars permit or require layers to use IOLTA for small sums that are to be held for relatively brief periods. The banks sends the interest on these IOLTA accounts directly to a foundation that uses the money to fund legal service programs for underrepresented people What goes into the client trust account? o Expense advance o Advance on attorney fees ABA Model Rule 1.15(a) requires an attorney to keep complete records of all clients money and property that comes into the attorneys possession o Records should be kept for a number of years specified by local rule (usually about five years) All 50 states have client security funds, a source of money that can be used to reimburse the hapless clients of dishonest lawyers. Foshee v. Lloyds, New York, (5th Cir. 1981) It was not abuse of discretion to permit enforcement of contingent fee contract according to its revised terms, which provided that attorneys would receive 100% of first $50,000 recovered. 50% of next $100,000, and 20% of any amount above $150,000, where arrangement was agreed to by attorneys solely at instance of clients, who felt such agreement would at least give them chance to pay off their creditors in full, and their was not slightest suggestion of any impropriety on part of attorneys. 35 | P a g e

Fee splitting Rule 1.5(e) Lawyers who are not in the same firm may divide fees only if: (1) the division is proportional to services performed by each or each assumes joint responsibility for the representation; (2) the client agrees to the arrangement, including the share each lawyer will receive, and the agreement is confirmed in writing; and (3) the total fee is reasonable. There are two ways to involve another firm: Referral generally receive 1/3 of the fee for a referral Associate another law firm and together participate in representation CHAPTER 6 COMPETENCE, DILIGENCE, AND UNAUTHORIZED PRACTICE Discussion Problems Rules 1.1-1.4, 1.8(h), 5.4-5.5(b) Unauthorized Practice of Law In Alabama- you have to receive consideration to be guilty of unauthorized practice of law In Alabama, this is a Misdemeanor 1. Lawyer Layton (educated at a famous law school that boasts a national legal education and graduated at the top of her class but licensed to practice law only in Arizona) responded to a legal inquiry made in an Internet chat room, which she frequents for her own pleasure. The inquiry was addressed to any lawyer willing to answer and regarded the viability of an action for legal malpractice in a divorce case that had been finalized 35 months prior to the inquiry in Rhode Island. Without doing any research regarding Rhode Island law, Layton responded (based solely on her national education) that the writer in fact had an actionable claim for malpractice and that the statute of limitations was four years, so she could still file the claim. In fact, the statute of limitations was 3 years in Rhode Island and she waited past 36 months in reliance on Laytons statement and missed her deadline to file. 1) Is Layton guilty of practicing law without a license in Rhode Island? It depends on how RI defines the practice of law. According to RI law she is guilty. In AL she would not be guilty because there was no consideration given therefore there was no relationship a. Is Layton subject to discipline for incompetence? YES she did not do any research RI cannot impose sanctions on her that will be effective in AZ, where she is licensed to practice, she is subject to discipline under Rule 5.5(a) Rule 5.5(a) In Rhode island, shed be guilty of this b/c the statute is different NO restrictions as to whether the atty is practicing law, or whether hes wearing his real estate agent hat, either way, he cant do this 36 | P a g e

b. Is Layton liable to Cushing for legal malpractice? YES, she could be prosecuted in RI, but she is unlikely to be extradited since this is probably a misdemeanor This brings up the issue of whether the inquirer was a client. Formality is not essential for the attorney-client relationship. If the client reasonably believes the relationship exists, this belief can very well be determinative. Be careful about answering casual questions. You may end up with an inadvertent client. B-Yes Do they have a lawyer/client relationship? Foramilty isnt required to create an atty/client relationship no need for an agreement Clients reasonable reliance they think that you are their atty.really bad if they act based on what you tell them B- c. Legal Malpractice1) Intentional tort 2) Breach of fiduciary duty 3) Breach of contract- warranty that atty will perform services etc.. 4) Unintentional tort- negligence Some duties can carry over to nonparties Std of care- established by statute in Alabama -Level of reasonable care, skill and diligence as like attys in similar circumstance, practices, locations etc. 2. Client Cameron (plaintiff) hired the A & B firm to represent her in a products liability action and paid them a $5,000 advance. The firm assigned Anson to her case but Anson left the firm without working on the case. The firm then reassigned the case to Benson who, due to a heavy workload, did not file the complaint for 10 months. When the case came to trail the judge took it under submission for 18 months, even though the state statute requires judges to decide cases within 90 days. Benson did not take measures to speed the judge up. The case was finally decided in Camerons favor, but she is mad that it took so long. a. Does Cameron have a valid claim for legal malpractice? NO no damages the most she lost was interest on the judgment and this was an insubstantial amount in this case. A good test for whether a malpractice claim could be maintained is whether you would take malpractice case on a contingent fee basis. In this case you would not. b. Rule 5.1 supervising attysif lawyer advises him to do this, or ratify the actions a. Law FIRMS cant be disciplined, only individual attys can be disciplined b. Judge Jergens- rule 3.5- cant ex parte communicate w/the judge but maybe call the other atty 37 | P a g e

c.

Judicial Inquiry Committee- can report the judge to this confidential committee- client can report him also

c. Is this an appropriate case for professional discipline? If so, who should be disciplined? a. Rule 1.3 Diligence A lawyer shall act with reasonable diligence and promptness in representing a client. b. Rule 5.1 puts on the management committee the responsibility of ensuring that the lawyers they are supervising are conforming to the Rules of Professional Conduct c. The lawyer cannot call the judge because he cannot have an ex parte communication with the judge about a case. d. How might you prompt the judge to act without violating a rule or offending the judge? You could call the clerk (or secretary if the judge has no clerk). Tell them about the wait and ask if the file could have been lost, etc. If there is no one like this to prompt, you could contact the attorney on the other side and work together to send a joint letter to the judge asking when you might anticipate a ruling. e. How do you report judges? Most states have a commission to investigate complaints against judges. These investigations can also happen without a complaint if the commission finds out something in another way. These complaints and investigations are confidential. If probable cause for a violation is found, it is referred to the court of the judiciary. The trial is public. f. Rule 8.3 mandatory duty to report misconduct by judges you can encourage the client to make the complaint 3. Lawyer Adams agreed to represent client Chandler against the airline for a personal injury and public humiliation suffered at the hands of an airline employee. Adams neglected the case and allowed the statute of limitations to run on the claim. Recognizing his mistake, he disclosed the error to the client and offered to pay Chandler $6500 out of his own pocket (that was $2000 more than the medical expenses had been). The client accepted, Adams wrote a personal check to the client and noted on the back that it was payment in full and released him from all claims. Adams also reported the settlement to the bar. Did he handle this properly? NO He is guilty of malpractice because he let the statute run. Malpractice is not predicated on disciplinary rules but on the duty of care to the client. He should have given her advice in writing to seek independent counsel. He must also give her an opportunity to obtain independent counsel. It would be wise to give her sometime (a few days) to think about the matter. It was not necessary to report this matter to the state bar. Rule 1.8(h) A lawyer shall not (1) make an agreement prospectively limiting the lawyers liability to a client for malpractice unless the client is independently represented in making the agreement; or (2) settle a claim or potential claim for 38 | P a g e

such liability with an unrepresented client or former client unless that person is advised in writing of the desirability of seeking independent counsel and is given a reasonably opportunity to seek the advice of independent legal counsel in connection therewith. Rule 1.8(h)(2) atty cant settle w/unrepresented client unless he has been given a letter waving seeking out independent counsel a. Rule 1.1 and 1.3 subject to malpractice b. Rule 1.8 cOI b/t Rule 1.9 COI b/t 2 clients Rule_______ COI b/t 4. Crampton lost use of his right leg when a nurse gave him improper medication at the hospital. Crampton consulted attorney Arlene; she told Crampton that she had never handled a medical malpractice claim before, but that she would do her best on Cramptons behalf. Ultimately Cramptons case went to trial and was lost. Then Crampton sued Arlene for legal malpractice, claiming these defects in Arlenes performance: a. Arlene failed to consult with any expert on hospital operations; an expert could have testified that the number of nurses at the hospital was insufficient to give proper care to all the patients. Malpractice one of the key issues was adequate staffing b. Arlene used only one expert medical witness at the trial, and the jury might have been more impressed if there had been several. No malpractice this is a mere error in judgment c. Arlene failed to find out whether there were any eyewitnesses (nurses or aides) around when Crampton received the improper medication. Malpractice she should have sent interrogatories (ask them to name all persons working or present that night) and requests for production (asking for medical records) d. Arlene failed to discover a State DH regulation setting the proper staff/patient ratio in hospitals. Malpractice this regulation could establish the standard of care and thus is relevant and material to the case Negligent in not getting expert testimony, and not finding witnesses at the time of the incident, negligent to not adhere to the Dept of health regulations

Is Arlene guilty of malpractice for taking the case in the first place? NO Rule 1.1 Comments 2 & 4 Do any of the four defects constitute good grounds for a legal malpractice claim? YES 39 | P a g e

Rule 1.1 Competence A lawyer shall provide competent representation to a client. Competent representation requires the legal knowledge, skill, thoroughness and preparation reasonably necessary for the representation. What about the venue of the case? Where are you going to file? Can you be liable for filing in a certain venue? No, not if you make a considered professional judgment. However, you should involve the client in this decision, after informing the client of all relevant factors. 5. Huffington (house counsel for Infoscope) contacted lawyer Lawyer Levitt with the following proposal: Infoscope had been sued for violation of a federal act (one which they have been sued under numerous times) in Levitts state. Levitt is offered $125,000 to dispose of the case (that would include all expenses, fees, litigation costs and settlement). Huffington offered further to provide Levitt with all the legal research, interrogatories, briefs etc...that have been used in past cases in order to significantly reduce the amount of work required to dispose of the case. Levitt will be entitled to keep whatever amount of the $125,000 that is left. Can Levitt represent Infoscope on the terms Huffington proposes? Rule 1.2 requires the attorney to follow the clients instructions. Here the lawyer is being constrained by the client to follow the clients methodology and the clients budget limit. This has some of the downsides of a contingent fee basis. You could end up with nothing. Any payment to the plaintiff comes out of this $125,000. Brewer said you could call the other attorney and tell him several cases like this have been done away with by summary judgment and offer a settlement that would leave you with some money. However, if it turns out that this case is different from the others and this case involves more and you want to withdraw, you may have some problem. Under 1.16(a) and (b) if the client asks you to do something illegal, or ill-advised, you dont have to take the case. A lawyer may reveal confidential information. this is an optional rule To prevent the client form fraud, financial, or other damage etc Sarbanes Oxley 6.Attorneys A, B &C are three shareholders in a law firm that is organized as a PC. The hardest working person in the firm is Daley, a non-lawyer. Her title is office manager and her duties include keeping the financial and billing records, supervising the office staff, and managing the client files. When she has time, she also helps the lawyers with research and she drafts routine documents. a. Ds brother and sister want to buy a small piece of real property as an investment and they want to keep the legal fees as low as possible. Over the years, D has picked up enough knowledge of real estate law to know exactly what to do. Would it be proper for D to do the legal work if C looks over her work to make sure that it is complete? Rule 5.3(c) This rule prohibits a lawyer from permitting a non-lawyer from practicing law without a license. Technically, there is a violation of the law because C has not made the professional legal decisions regarding this transaction (e.g. how will they hold the 40 | P a g e

property, terms of the note, etc.). D may do the work, but it must be at the direction of the lawyer. Checking the work over once it is done is not enough. The document may be fine and have no errors, but may not be what the client would have wanted. (The problem is with making the professional decisions not typing the document.) b. One of Ds friends selected A, B &C to represent him in a major matter, thanks largely to his friendship with D. May the firm pay D a bonus equivalent to 10% of the fees earned in this matter? NO Rule 5.4(a) A lawyer is not to share fees with a non-lawyer!!! Cannot allocate a percentage of the fee to a person who refers the client. Firms can pay bonus from the aggregate of fees collected on all their cases. c. The firm proposes to set up a retirement program that will be funded in part by fees earned by the lawyers. Would it be proper to include D as a participant in the program? YES Rule 5.4(a) Professional Independence of a Lawyer A lawyer or law firm shall not share legal fees with a nonlawyer, except that: (3) a lawyer or law firm may include nonlawyer employees in a compensation or retirement plan, even though the plan is based in whole or in part on a profit-sharing arrangement d. The states professional corporation statute requires corporate officers to be shareholders. Bell is officially named treasurer of the PC, but in fact D does all the financial work. May the attorneys sell D a token number of shares and name her as the corporate treasurer? NO Rule 5.4(d) A lawyer shall not practice with or in the form of a professional corporation or association authorized to practice law for a profit, if: (1) a nonlawyer owns any interest therein, except that a fiduciary representative of the estate of a lawyer may hold the stock or interest of the lawyer for a reasonable time during administration; (2) a nonlawyer is a corporate director or officer thereof or occupies the position of similar responsibility in any form of association other than a corporation; or (3) a nonlawyer has the right to direct or control the professional judgment of a lawyer LEGAL MALPRACTICE The Relationship Between Legal Malpractice and Disciple by the Bar Legal Malpractice = attorneys civil liability to a client or other injured person for professional misconduct or negligence In a negligence action, the trier of fact may consider the defendant lawyers breach of such a statute or rule as an aid to understanding and applying the standard of care, provided that the statute or rule was designed to protect people in plaintiffs position. Theories of Legal Malpractice Liability Several legal theories are available: o Intentional tort 41 | P a g e

o Breach of fiduciary duty o Breach of contract o Unintentional tort ordinary negligence E.g. failed to meet deadlines To whom does an attorney owe a duty of care? o An attorney owes a duty of care to a client A person can become a client simply by asking the attorney for legal help, if the attorney does not decline to give the help, and if the attorney knows or should know that the person will reasonably rely on the attorney to give the help o An attorney owes a duty of care to a non-client in four situations: Prospective client Rule 1.18 A prospective client is a person who discusses with a lawyer the possibility of forming a client-lawyer relationship with respect to a matter. Invited reliance E.g. bank relying on a title of opinion Non-client is intended to benefit E.g. intended beneficiary of a will Breach of fiduciary duty by client What is the standard of care? o General practitioner the standard of care is the skill and knowledge ordinarily possessed by attorneys under similar circumstances o Specialist the attorney must exercise the skill and knowledge possessed by attorneys who practice that specialty o The relevant geographic area for defining the standard of care is the jurisdiction (normally a state) in which the lawyer rendered the questioned legal services. Lawyers in urban areas and rural areas are held to the same standard. Elements: duty, breach of duty, actual cause, proximate cause, damages Breach of the duty of care o Lawyers are not liable for mere errors in judgment. o The judgment must be a well-informed judgment, not one made in ignorance. o A lawyer is expected to know the settled principles of law. o If a principle of law is unsettled and open to debate, the attorney is expected to do reasonable research and to make an informed decision as to a course of conduct based upon an intelligent assessment of the problem. o An attorney can be held liable for failing to conduct a reasonable fact investigation, or failing to find and interview key witnesses, or failing to consult with appropriate experts, or failing to discover pertinent statutes, regulations, and the like. Actual cause o Actual cause usually means proof that the injury would not have happened but for the defendants negligent act

42 | P a g e

o Preponderance of the evidence o A malpractice trial sometimes becomes a trial within a trial. o When a but for analysis is inadequate to determine actual cause, a court can use the substantial factor analysis. Proximate cause o A malpractice plaintiff must prove, not just actual cause, but proximate cause. Damages o Direct damages for the immediate, natural, and anticipated consequences of the wrong o Consequential damages damages for loss that flows indirectly but foreseeably from the defendants negligence

Defenses to Legal Malpractice Claims The attorney reasonably believed that the action was required by a law or a legal ethics rule Comparative or contributory negligence Assumption of the risk and failure to mitigate damages Statute of limitations: o The statute of limitations ordinarily does not run on a clients malpractice claim while the lawyer continues to represent the client in the matter at hand or a substantially related matter o Does not start to run until the lawyer discloses the supposed malpractice to the client, or the facts that the client knows or reasonably should know clearly indicate the malpractice occurred o Does not start to run until the alleged malpractice significantly injures the plaintiff Vicarious Liability A law firm is civilly liable for injuries cause by an employee or principal of the firm who was acting in the ordinary course of the firms business, or with actual or apparent authority. If a law firm is organized as a partnership without limited liability, the general law of partnership makes each partner liable jointly and severally with the firm. New entities professional corporations, limited-liability general partnerships, and limited-liability companies law firm generally remains vicariously liable for injuries cause by an employee or principal who was acting in the ordinary course of the firms business, or with actual or apparent authority. However, the principals of the firm are generally not personally liable for negligence or misconduct in which they did not participate personally or as supervisors. Malpractice Insurance The ABA Model Rules do not require lawyers to carry malpractice insurance. Most modern American lawyers regard malpractice insurance as an essential. Policies differ dramatically in their features: 43 | P a g e

o Occurrence policies cover the lawyer for acts or omissions made during the policy term, regardless of when the claim is asserted no longer available o Claims made policy covers the lawyer for unforeseen claims made during the policy period, no matter when the act of omission occurred o Supplemental prior acts coverage prevents gaps in insurance coverage o Liability policies generally require the insurer to defend the lawyer against covered claims o The higher the policy limits, the higher the premium. o Typically, defense costs are included in the policy limits. o The size of the deductible can also make an important difference in the cost of a policy. o Most policies provide for a per claim deductible, but a few specify an aggregate deductible during the policy term. o Policies differ as respects the persons who are covered. o Policies vary in the kinds of acts and omissions they cover. o All policies contain exclusions. The number and breath will affect the cost of the insurance. THE ETHICS OF SECOND-RATE LEGAL SERVICE When the client wants the lawyer service, but insists on a budget too low to let the lawyer provide the service competently and diligently, some authorities require the lawyer to decline the representation, or to withdraw if the representation has already begun. MULTI-JURISDICTIONAL AND MULTI-DISCIPLINARY PRACTICE Multi-Jurisdictional Practice ABA Model Rule 5.5 the unauthorized practice of law encompasses both nonlawyers and lawyers licensed in other states A lawyer licensed and in good standing in one jurisdiction, may practice, temporarily, in another jurisdiction when the lawyer o (1) works in association with a lawyer licensed in the host state; o (2) represents clients in, or in participation in, an arbitration, mediation, or other alternative dispute resolution proceeding; o (3) performs non-litigation work that arises out of, or is reasonably related to, the lawyers home-state practice; and o (4) provides litigation-related services in a state where the lawyer expects to be admitted Multi-Disciplinary Practice (MDP) A MDP is a partnership, professional corporation, or other association or entity that includes lawyers and nonlawyers and has one, but not all, of its purposes the delivery of legal services to a client(s) other than the MDP itself or that holds itself out to the public as providing nonlegal, as well as legal, services. 44 | P a g e

Currently, ABA Model Rule 5.4 prohibits the creation of MDPs o Rule 5.4(a) prohibits lawyers from sharing fees with non-lawyers o Rule 5.4(b) prohibits the creation of a partnership between a lawyer and a non-lawyer if the partnership will engage in the practice of law The growth of MDP shows no sign of abating

Supplement In re Jackson, (Louisiana 2003) Facts: Mr. Brown contacted Attorney Jackson by telephone regarding instituting a medical malpractice proceeding arising from the death of his mother. Brown forwarded the paperwork and Jackson said he would look at it when he got a chance. Brown and Jackson commuted regarding the case several times. As the deadline for filing approached, Jackson stopped communicating with Brown. Suit was not filed till the deadline passed. The suit was then dismissed as being prescribed. Result: Jackson is suspended from the practice of law for six months, three of which shall be deferred. Following this he will be placed on probation for one year, subject to completion of the Louisiana State Bar Associations Ethics School program. Rationale: o An attorney-client relationship existed. o A layperson hires an attorney in order to have the benefit of the attorneys skills, which includes the attorneys ability to determine when the claim must be filed. o A period of suspension is warranted to impress upon Jackson that this court will not tolerate disregard by an attorney of the interests of his client. CHAPTER 7 CONFIDENTIAL INFORMATION Discussion Problems Rules 1.2(d), 1.6, 1.8(b), 1.9(b), 3.3, 3.4(a), and 4.1 *Confidentiality is an ethical rule, much broader than the atty-client privilege *Attorney-Client privilege is a rule of evidence applies only in Judicial Proceedings 1. Note: attorney client privilege gives the client the right to prevent witnesses from revealing confidential communications between the attorney and client or the attorneys agents. Client is the holder and the privilege applies whenever the twin powers of subpoena and contempt compel the giving of information. How does the attorney-client privilege differ from the attorneys ethical duty to preserve the clients confidential information? Consider the following situations: a. While standing around a PTA potluck supper, lawyer L gossips with a friend about the reasons Ls client V wants to divorce her husband. 45 | P a g e

Does the attorney-client privilege apply at the potluck? NO not a judicial proceeding; applies specifically when the government has the powers of subpoena and contempt. Only applies when the government is seeking to compel evidence. **Any time you see a government proceeding, be cautions** Does the ethical duty? YES. Rule 1.6. Applies always except in a situation where the attorney-client privilege applies. (This would be a violation of confidentiality under Rule 1.6; Can have discipline by an ethics committee; if your client suffers damages the client can bring a civil suit against you) If the attorney was gossiping and violated the privilege/confidentiality, the attorney would be subject to disciplinary sanctions and the subject to a civil suit by the client for breach of fiduciary duty (especially if the client could prove that she suffered damages from the lawyers gossiping). b. Lawyer L is defending client X in a drunk driving case. Through her own investigations, L learns from a talkative bartender that X stops in for several double martinis every night after work. Does the attorney-client privilege protect that information? NO no communication between the attorney and the client If not, is L free to reveal it to whomever she wishes? NO ethical duty still applies cannot reveal any information regarding client 1.6(a) c. Client Y tells lawyer L in confidence that he want to purchase Blackacre to build a new shopping center. Acting as an undisclosed principle, L instructs her agent to buy Black Acre hoping to turn a quick profit on resale to Y. Has L violated the attorney-client privilege? NO privilege is a rule of evidence Has she violated the ethical duty? YES Rule 1.8(b) A lawyer shall not use information relating to representation of a client to the disadvantage of the client unless the client gives informed consent, except as permitted or required by these Rules Example: 1.2(d), 1.6, 4.1(b) d. Suppose instead that L buys Greenacre (GA), which adjoins BA, knowing that it will triple in value when Y builds the shopping mall on BA.

46 | P a g e

Has L violated the ethical duty? There is not a rule on this, but as a practical matter it may undermine the clients trust it could turn out to work a disadvantage of the client and be a Rule 1.8(b) violation because the lawyer may be in competition with the client for tenants agency law may apply and require disgorgement should not use fiduciary relationship for personal profit; Self-dealing problems here (recourse dont get to keep the profits) e. Client Z told L in confidence: Yesterday I intentionally burned down my barn because I need the fire insurance money. I want you to represent me in collecting on my insurance policy. L declined to represent Z, who then hired lawyer M to pursue the claim. (Z did not tell M that he burned down the barn.) The insurance company refused to pay, asserting that Z burned down the barn to get the insurance money. At Zs trial, the insurance company called L to the witness stand and asked: what did Z tell you about burning the barn? 1. Should the court sustain Zs claim of attorney-client privilege? Normally attorney-client privilege would apply, but not where the attorneys services are being sought to aid in a crime. If ordered Can only reveal what is necessary to comply with a court order (talk to client first) Under Rule 1.6(6) we must comply with court order or law. Confidentiality is an ethical issue that is enforced by the disciplinary board, not the court.

Notice that this is an EXCEPTION to Rule 1.6

In a court proceeding, the applicable body of law that applies is the rules of evidence; NOT the model rules 2. When Z left Ls office, should L have warned the insurance company that Z was planning to file a fraudulent claim? NO confidentiality Rule 1.6(b) (2) this would be revealing information relating to the representation of a client but the client has not employed their lawyers services in the furtherance of the crime/fraud. Rule 1.18 (this rule deals w/ a prospective client that we dont later represent --he was a prospective client)

47 | P a g e

2. Dorman is in jail awaiting trial for the first-degree murder of a young girl. Attorney Anthony is appointed to defend D. D tells A in confidence that he killed not only that girl, but also two other young girls. D tells A where the other bodies are hidden and A goes there and determines that D is telling the truth. No one else knows that the other two girls are dead; they are being sought as runaways. What should A do? He may not reveal what he has learned from his client (Rule 1.6(a)). He may counsel his client (Rule 1.2(d)) plea bargain, etc. In order to use 1.6(b)(1) there must be imminent death or substantial bodily harm (in the FUTURE). If you find a girl that is still alive, then attorney can disclose under 1.6(b); do not HAVE to disclose, but MAY disclose. In real life, the attorney didnt tell for ten months. State want after him on a public health statute. He lost his law license. (Notice that this is 30 years before the ABA model rule was passed) 1.4(b) makes it incumbent to communicate with your client that the ethical duties may require disclosure in certain situations. (generally comes up in criminal representation) **3.3 trumps 1.6 (per Comment 15 to 1.6) 3. On the afternoon of 8-11-03, a woman walked into your law office, stated her name, and said in confidence: I am the driver the police are looking for in that fatal hit and run accident last week. You agreed to represent her, and you advised about the wisdom of surrendering to the police, but she rejected your advice. The police have never discovered the identity of the hit and run driver. Just prior to the expiration of the statute of limitations, the parents of the hit and run victim file a wrongful death action against Jane Doe defendant. Acting on a hunch, the parents lawyer has subpoenaed you as a deposition witness and has asked for the names of all the persons who consulted you on the afternoon of Aug. 11th. What should you do? Challenge the subpoena. Generally the identity of a client is not privileged because it is not a communication, but the clients identity in this case is confidential (because revelation of the identity would reveal the essence of the communication). Source of the fee would also be privileged in that situation Protected under Attorney-Client privilege. 4. Your law practice includes some criminal defense work. A few minutes ago, one of your steady clients stormed into your office, waiving a pistol and announcing that he just killed his probation officer. You have urged him to surrender to the authorities, but he has refused, stating that they will catch him sooner or later and he wants to enjoy the last of his freedom. He has laid the pistol on your desk and is about to walk out of the office without it. What should you do? Do not keep the gun. Tell the client to take it with him. If the client still leaves the gun, you will have to turn it over to police eventually Rule 3.4 (a) and (b) an attorney cannot conceal or destroy evidence or counsel the client to conceal or destroy evidence tell him 48 | P a g e

that if he has the weapon on him that will be damning if he is caught with it, but if he destroys it and that is found out it will also hurt him and will be an additional crime If he leaves the gun, still not protected. NOT a communication. 5. Your client, Enos Furman, is in the business of leasing expensive equipment to farmer. First, he arranges long-term equipment leases with the farmers. Then he borrows money from the bank to purchase the equipment; he uses the long-term leases as security for the bank loans. You have acted as Fs lawyer in ten of these lease-loan transactions over the past two years. Today he revealed to you, in strict confidence, that some of the leases he used in those transactions were fakehe forged them and thus tricked the banks into lending him the money which he has long since spent. He has solemnly promised you that he will never so that again, and he had asked you to serve as his lawyer in a series of new lease-loan transactions. What are you ethical obligations in this situation? Counsel client about the potential for difficulty he may face and suggest he allow you to contact the bank to work out a payment plan. The client has criminal liability theft, false pretenses, forgery, etc. The client has civil liability and the potential for punitive damages since the act was willful. May withdraw and disavow. [Rule 1.16] Can do a noisy withdrawal since he has used your services. Under the new amendments you could also just call the bank and reveal what he has done (since he services have been used to perpetrate the fraud).

Class Notes: Attorney-client privilege deals with evidence court setting only Confidentiality ethical issue Remember that as a defense attorney we do not owe the client a victory. We owe the best representation we can give.

Washington v. Olwell, (Wash. 1964), p.166 Facts: Gray and Warren engaged in a fight, which resulted in Warren being mortally wounded by knife wounds. Gray admitted to stabbing Warren, but told police he was not sure what happened to the knife. Grays attorney later came to be in possession of the knife. The coroner issued a subpoena duces tecum. The attorney refused to comply (claiming this is a violation of an attorney-client relationship). The attorney was found to be in contempt of court. Issue: May an attorney refuse to produce, at a coroners inquest, material evidence of a crime by asserting the attorney-client privilege or by claiming the privilege against self-incrimination on behalf of his client? Result:

49 | P a g e

o On the basis of attorney-client privilege, the subpoena duces tecum is defective on its face because it requires the attorney to give testimony concerning information received by him from his client in the course of their conferences. o The clients 5th Amendment privilege against self-incrimination could not serve as a shield for the attorney. The privilege against selfincrimination is personal to the client and must be claimed by the client alone. Rules: o To be protected as a privileged communication, information or objects acquired by an attorney must have been communicated or delivered to him by the client, and not merely obtained by the attorney while acting in that capacity for the client. o Communications concerning an alleged crime or fraud, which are made by a client to the attorney after the crime or the fraudulent transaction has been completed, are within the attorney-client privilege, as long as the relationship of attorney and client has been established. o The court must balance the attorney-client privilege and the public interest in criminal investigation. o The attorney should not be a depository for criminal evidence. After a reasonable period (for testing), the attorney should, as an officer of the court, on his own motion turn the evidence over to the prosecution. It is a crime to destroy or alter evidence o The state, when attempting to introduce such evidence at trial, should take extreme precautions to make certain that the source of the evidence is not disclosed in the presence of the jury and prejudicial error is not committed.

People v. Meredith, (Cal. 1981), p. 170 Facts: Scott and Meredith murdered and robbed of Wade. Scott had revealed to his attorney where to find the victims wallet. Based on this information, the defense attorneys investigator located the wallet. After examining the wallet the defense turned it over to police. To support the theory of conspiracy, the prosecution sought to show the place where the victims wallet was found. Issue: Whether under the circumstances of this case, the defense attorneys investigators observation of the location of the wallet, the product of a privileged communication, finds protection under the attorney-client privilege. Result: The prosecution was entitled to present evidence to show the location of the wallet in the trashcan. Rules: o An observation by defense counsel or his investigator, which is the product of a privileged communication, may not be admitted unless the defense by altering or removing physical evidence has precluded the prosecution from making that same observation.

50 | P a g e

o Whenever defense counsel removes or alters evidence, the statutory privilege does not bar revelation of the original location or condition of the evidence in question. [the location itself is evidence] o If defense counsel leaves the evidence where he discovers it (merely observes the evidence), his observations derived from privileged communications are insulated from revelation. o In offering the evidence, the prosecution should present the information in a manner which avoids revealing the content of attorney-client communications or the original source of the information. [As an attorney you should file a motion in limine and stipulate that this piece of evidence was found at the particular location at the particular time.] EXCEPTIONS TO THE ETHICAL DUTY OF CONFIDENTIALITY Client Consent or Implied Authority o Rule 1.6(a) allows a lawyer to reveal a clients confidential information if the client has given informed consent, or it the nature of the representation impliedly authorizes the lawyer to reveal the confidential information. Self-Defense o Rule 1.6(b)(3) allows a lawyer to reveal a clients confidential information in self defense Defend against a claim of legal malpractice or ineffective assistance of counsel Defend against a civil or criminal charge that the lawyer was involved in the clients wrongdoing To obtain relief against a client who has breached a fee agreement or the like o Rule 1.6(b)(2) allows a lawyer to reveal a clients confidential information in order to get legal advice about complying with the rules of legal ethics Court Order or Other Law o Rule 1.6(b)(4) allows a lawyer to disclose a clients confidential information where that is necessary to comply with a court order or with some other law Prevent Future Harm o Attorney-client privilege does not protect communication in which the client seeks the lawyers services to aid in the planning or commission of an ongoing or future crime or fraud o Under the ABA Model Rules, a lawyer may reveal a clients confidential information to prevent reasonably certain death or substantial bodily harm (Rule 1.6(b)(1)) An Almost Exception Noisy Withdrawal o Rule 1.2(d) prohibits a lawyer from assisting a client to commit a crime or fraud

51 | P a g e

o Rule 4.1(b) requires the lawyer to disclose material facts to a third person when disclosure is necessary to avoid assisting a criminal or fraudulent act by a client, unless disclosure is prohibited by Rule 1.6 o ABA ethics committees noisy withdrawal provision: The lawyer must withdraw from any part of the representation that directly of indirectly assists the client in perpetrating the fraud. The lawyer may withdraw from all representation of the client, and she must withdraw from all representation if to continue would be likely to mislead the fraud victim into thinking that all is well. When she withdraws, the lawyer may disavow any of her work product to prevent the client from using it to carry out the fraud. This disavowal is proper, even if the effect is to reveal the clients fraud. If the fraud is completed, and the client is not planning to use the lawyers services or work product to commit further fraud, the lawyer may withdraw but must not disavow her work product. In other words, she may withdraw, but not noisily. Dietz v. Doe, (Wash. 1997) Facts: Personal representative of drivers estate brought action to compel attorney to disclose name of putative client who allegedly was involved in fatal accident. Result: Remanded o The trial court must address whether the unknown driver was the attorneys client for purposes of attorney-client privilege o The trial court must determine the nature of the attorneys consultation with the putative client o The trial court must determine whether the putative client waived privilege Rules: o The general rule is that the identity of an attorneys client is not privileged. o There is a legal advice exception to the general rule: where the disclosure of the clients identity necessarily reveals the essence of the communication between the attorney and the client, the clients identity need not be disclosed. 8 part test: The client must have sought legal advice From the attorney in his or her capacity as an attorney The communication must have been made in order to obtain legal advice In confidence By the client The client must wish to protect the clients identity From disclosure

52 | P a g e

The protection must not have been waived o The attorney-client privilege can ordinarily be waived only by the client, to whom the privilege belongs. However, when the attorney is authorized to speak and act for the client on particular matters, disclosures by the attorney that are within the scope of that authority waive the privilege to the same extent as disclosures by the client. Swindler & Berlin v. United States, (U.S. 1998) Facts: An attorney made notes of an initial interview with a client, Vincent Foster Jr., shortly before the clients death. The government, represented by the Office of Independent Counsel, sought to obtain these notes for use in a criminal investigation. The attorney and his law firm moved to quash grand jury subpoenas. Result: The attorney-client privilege survives the death of the client

CHAPTER 8 CANDOR IN LITIGATION Discussion Problems ABA Model Rules 1.6, 3.3, 3.4, and 4.1 Rule 3.3 trumps Rule 1.6 Rule 4.1 does not trump Rule 1.6 (1.13 also trumps 1.6, but it is a may not a must) The only MUST DISCLOSE (of privileged information) is when your client has lied to the court 1. Suppose you represent the defendant in a diversity case that is in trial in the US District Court for the SD of NY. The applicable state law is that of the state of NY. One disputed legal issue is vitalin a malpractice what is the appropriate standard of care for a doctor of veterinary medicine who holds herself out as a specialist in ruminant epidemiology? The plaintiffs lawyer has failed to cite a very recent appellate decision that would support the plaintiffs position. So far as you know, the trial judge is unaware of the decision, but you know about it because it was reported in the current issue of US Law Week. In which of the following situations would you have an ethical duty to call the decision to the trial courts attention? Rule 3.3(a)(2) A lawyer shall not knowingly: fail to disclose to the tribunal legal authority in the controlling jurisdiction known to the lawyer to be directly adverse to the position of the client and not disclosed by opposing counsel a. Suppose that it were a NY Court of Appeals (highest NY court) decision in a veterinary malpractice case? Duty to disclose it is a controlling authority and it is directly adverse b. Suppose that it were an Arizona Supreme Court decision in veterinary malpractice case? No duty to disclose not in a controlling jurisdiction c. Suppose it were a NY Court of Appeals decision in a legal malpractice case involving a lawyer who held himself out as an expert in Robinson53 | P a g e

Patman price discrimination litigation? No duty to disclose not directly adverse; this is not a legal malpractice claim it is a veterinary malpractice claim Are there any tactical reasons to go beyond what the ethical rules require? Yes it is persuasive authority, if you bring it up and distinguish it before anyone else finds it you may reduce the harm that it could cause (defang it) as long as you make a considered judgment about this, either way you will probably be safe from legal malpractice Remember, dont try to oversell your position. 2. Suppose you represent the defendant at the trial of a negligence case. The plaintiff has engaged in extensive discovery but has not discovered witness X, an impartial 3d party who saw the accident clearly. Zs testimony would establish that your client was at fault. At the trial, plaintiffs case in chief is insufficient to get the case to the jury, but the defect could be cured by Xs testimony. Plaintiff is about to close her case in chief. When she does, it will be time for you to move for a directed verdict. You know that X lives nearby and is available to testify. What will you do? Nothing no duty in state court Rule 3.4, Comment 1 and Rule 4.1, Comment 1 However, FRCP 26(a)(1): duty to disclose names of witnesses (automatic disclosure) The other side should have asked for names of witnesses in interrogatories if you are asked there is a duty to respond if it may lead to any relevant information Exception ex parte hearing must give all relevant facts because this is not an adversary proceeding Rule 3.3(d) other side is not there to present the facts Obligation to Disclose Rule 26 (name of each individual who has discoverable info, documents, tangible things in your possession that you are using to support your position) General Rule Unless it is an ex parte proceeding or Federal Rule of Civil Procedure 26 applies, there is no duty to disclose harmful facts 3.3 applies ONLY when you are in a proceeding before a tribunal; also applies to an ancillary proceeding (like a deposition) lawyer must take remedial measures to correct the misrepresentation If client does not rectify the misrepresentation then disclosure is not the first option although it is allowable under 1.6 Must seek to withdraw if representation will result in a violation of the rules of professional conduct (Not a lot of support for this rule) If withdrawal is not permitted under applicable law (1.16(c) trumps 1.16(a)) then must disclose under 1.6 (3.3 trumps 1.6 Last resort after reasonable remedial measures) 1.6(b) MAY 3.3 trumps 1.6 54 | P a g e

1.13(c) trumps 1.6 even though it is a may Noisy withdrawal Lawyer withdraws on the eve of trial, etc. then the other side will know something is up. Lawyer must disclose the misrepresentation. Does NOT apply when drafting a contract, etc. Only applies to proceedings in front of a tribunal or an ancillary proceeding related to a tribunal 4.1 applies to other parties outside of the tribunal. Have a 4.1 situation when there is a lie or misrepresentation of the facts by attorney or client. Attorney has an obligation to disclose as long as it is not prohibited by 1.6 (basically, look at the exceptions of 1.6). Obligation is limited to your obligations under 1.6 1.6 trumps 4.1 3. Suppose that you are counsel for the patentee of a U.S. patented prosthetic arm that enables an amputee to function almost perfectly. Your client is the defendant in a federal declaratory judgment action brought by a competitor to have the patent declared invalid under 35 U.S.C. 102(b). That statute makes a patent invalid if the invention was in public use or on sale in the U.S. more than one year prior to the date of the U.S. patent application. The patent application was dated April 15, 2005. The plaintiff took your clients deposition and questioned him as follows: Q: What date was the patented prosthetic arm first sold in the U.S. A: August 21, 2004 Q: How do you know that so quickly and certainly? A: Because when you guys sued, I checked the date on the hang-tag on the sample prosthesis in our sample vault. That date tells when the item was first sold anywhere in the world. We put a dated hang-tag on a sample of everything we invent, and we store the sample in the sample vault, so we will have an accurate record. Q: How do we know that you put the right date on it? A: Because we put it on there with pen and ink, and we are not slimeballs like some people I can think of. After a few more questions, the deposition was adjourned for the lunch hour. During lunch, you admonished your client not to call the other side slimeballs. He responded:

55 | P a g e

They are slimeballs, and you have to fight slime with slime. Originally the hang-tag said 3/21/04, but I doctored it by turning the 3 into an 8 to make it say August 21, 2004. Pretty slick, no? What must you do now? Rule 3.3(a)(3) A lawyer shall not knowingly offer evidence that the lawyer knows to be false. If a lawyer, the lawyers client, or a witness called by the lawyer, has offered material evidence and the lawyer comes to know of its falsity, the lawyer shall take reasonable remedial measures, including, if necessary, disclosure to the tribunal. A lawyer may refuse to offer evidence, other than the testimony of a defendant I a criminal matter, that that lawyer reasonably believes is false. Under the language of 3.3, Comment 1, Rule 3.3 applies to ancillary proceedings such as depositions so it applies here. Rule 3.3(a)(3) imposes a duty on a lawyer in this position to take reasonable remedial measures. According to Comment 10, The advocates proper course is to remonstrate with the client confidentially, advise the client of the lawyers duty of candor to the tribunal and seek the clients cooperation with respect to the withdrawal or correction of the false statements or evidence. If that fails, the advocate must take further remedial action. If withdrawal from the representation is not permitted or will not undo the effect of the false evidence, the advocate must make such disclosure to the tribunal as is reasonably necessary to remedy the situation, even if doing so requires the lawyer to reveal information that otherwise would be protected by Rule 1.6. It is then up to the tribunal to decide what to do. 3.3 (must) trumps 1.6 (may) make disclosure under this situation. 4. Suppose that you are in-house counsel for a drug company that has been sued in a state court products liability case and you have hired outside attorney A to defend the company. You are technically responsible for all litigation matters, but do not second-guess outside counsel. In this case, the P claims to have gotten bleeding ulcers form taking Luxair, a drug made and sold by the company as a remedy for male pattern baldness. The active ingredient in the drug is phlogestin. P alleges that the company knew all along that the drug creates a grave risk of stomach ulcers in males over 40. P demanded production of a host of documents, including all documents relating to Luxair and stomach ulcers. In the files pertaining to a different drug product that also contains phlogestin, the paralegals found a smoking guna packet of research reports that show that when the D put Luxair on the market, it knew beyond a doubt that phlogestin significantly increased the risk of ulcers in males over 40. A plans not to produce the research reports because they were not in the files pertaining to Luxair and they do not mention Luxair. Do you agree with his plan? The law is not explicit on this point. Look at the terminology of the request (there may be a definition section) Look at the rules requiring production (Ethical rules and Rules of Civil Procedure 30 and following) Look at practical considerations 56 | P a g e

We may rely on the precise wording (letter) of the request. There may be practical considerations such as a judge that does not like technical evasions. You will generally not get in trouble however you decide here. Might want to rely on the advice of the outside counsel.

Rule 4.1(b) In the course of representing a client a lawyer shall not knowingly fail to disclose a material fact when disclosure is necessary to avoid assisting a criminal or fraudulent act by a client, unless disclosure is prohibited by Rule 1.6 Rule 1.6(b)(1) and depending on the state law possibly (6) 5. Suppose that you are the defense lawyer for Decker who is charged with firstdegree murder. He has told you that he is innocent and that he was miles away from the scene of the crime, playing cards with three friends. You have interviewed the three friends, and they confirm his story and you plan to use all three as well as D as witnesses at Ds jury trial. a. Suppose that ten weeks before the trial, your investigator hands you information that clearly shows, beyond any doubt, that Decker and his friends are lying and that D did commit the murder. When you confront your client he tells you that you are his lawyer not his jury. He says he want to testify and to have his friends testify. He wants the jury to decide if he is guilty. What should you do? Rule 3.3(a)(3) Do we know this is false? If we do not know this is false, we do not have to worry A defendant has a right to testify A lawyer has the right under Rule 3.3(a)(3) to refuse to offer evidence, other than the testimony of a defendant in a criminal matter that the lawyer reasonably believes is false encourage the client not to perjure himself and warn him of the consequences of perjury You can try to withdraw (1.16(a)(1) MANDATORY WITHDRAWAL I have ethical concerns what you would say to the judge; if the judge permits your withdrawal you have no further obligations with regard to clients testimony because you are no longer the attorney, guy is no longer your client, and lawyer has not called client as a witness) If you are not permitted to withdraw, must tell the judge under 3.3(a)(3) as long as there is not a state or local law that trumps How would a lawyer be held to KNOW (higher standard than reasonably believe) that the client is falsely testifying? A lot of jurisdictions dont agree with this!

Reasonable inquiry Past experience Cant stop your client from telling you something Cant be blissfully ignorant

57 | P a g e

Cant refuse to connect the dots

Know Lawyer MUST not offer false evidence; CAN prevent criminal D from testifying

Reasonably Believe Lawyer CANNOT keep client off the stand (CRIMINAL DEFENDANT) if the client wants to testify

b. Suppose the same facts, except that you get to get the information and confront D ten minutes before you are to begin presenting the defenses case-in-chief. First ask for a recess because you need some time talk to your client counsel the client about the consequences of perjured testimony tell him he has the right to testify, but you must refuse to put on the testimony of the other witnesses if you know they will offer false testimony the judge is unlikely to allow you to withdraw have to tell the court Be suspicious of any answer to MC that gives you disclosure as the first thing c. Suppose that you do not get the information until ten minutes after you have presented the testimony of D and his friends. When you confront D, he admits to the crime but says the jury should decide whether or not they believed him and his friends. He wants you to leave it as is. What should you do? Encourage D and his friends to recant their testimony, and tell them if they do have an ethical obligation to take remedial measures, try to withdraw (not a mandatory withdrawal, but still pretty noisy); only suffice if withdrawal is remedial in nature (noisy enough that it undoes the fraud), or disclosure to the tribunal, the decision is then the judges to make Rule 3.3(a)(3) testimony has already been offered unlike (a) and (b) Judge will probably just tell jury to disregard testimony if withdrawal is noisy d. Suppose that you do not get the information until ten weeks after the jury has acquitted D. When you confront D he confirms that he murdered the guy. But obviously the jury believed me and my friends, so leave well enough alone. What should you do? There is no longer a duty to disclose because the proceedings have concluded Rule 3.3(c) the duty to disclose extends only to the conclusion of proceedings e. Suppose that the information you receive from the investigator leaves some small doubt. When you confront D, he sticks to his story about the card game. Does that change your answers to a-d? Yes f. Compare your answers to questions a-e with the answer to problem 5 in Chapter 7. Are you consistent? If not, what accounts for the difference? 58 | P a g e

Class Notes: There may be a greater duty to reveal information to the court than to the other party. THE TRILEMMA: TRUST, CONFIDENTIALITY, AND CANDOR We are to seek the clients trust and to find out everything the client knows about the case. We are to preserve our clients confidential information (except in very limited situations). We are to act with candor, to refrain from presenting evidence we know is false, and (in some situations) to reveal our clients frauds. The Old ABA Model Code Position The Narrative Testimony Position Monroe Freedmans Position Sometimes it is proper for a criminal defense lawyer to present testimony that she knows is perjurious. The ABA Model Rules Position Nix v. Whiteside (U.S. 1986) o Issue Whether Whiteside was deprived of his right to effective counsel when his counsel told him if he testified to a story the counsel believed was false, the counsel would try to withdraw and (failing that) would tell the judge the story was false. o The Court held that Whiteside was not deprived of the effective assistance of counsel Robinsons conduct fell within the wide range of acceptable responses to proposed client perjury People v. Johnson (Cal. App. Ct. 1998) Facts: Johnson was convicted of numerous violent sexual offenses, kidnappings, and robberies. He contended he was denied his right to testify in his own defense. Result: The court erred in denying Johnson his constitutional right to testify, but he error was harmless Rationale: o The U.S. Supreme Court has explicitly held a criminal defendant has a constitutional right to testify on his own behalf. The right to testify is essential to due process of law in a fair adversarial process and is protected by the 5th, 6th, and 14th Amendments. o A criminal defendant has the right to take the stand even over the objections of his trial counsel. o Of the various approaches to a clients stated intention to commit perjury, the court believes the narrative approach represents the best accommodation of the competing interest of the defendants right to testify and the attorneys obligation not to participate in the 59 | P a g e

presentation of perjured testimony since it allows the defendant to tell the jury, in his own word, his version of what occurred, a right which has been described as fundamental, and allows the attorney to play a passive role. o Cant rely on the narrative testimony in closing. ABA Formal Opinion 93-376 (1993) A lawyer in a civil case who discovers that her client has lied in responding to discovery requests must take all reasonable steps to rectify the fraud, which may include disclosure to the court. In this context, the normal duty of confidentiality (Rule 1.6) is explicitly superceded by the obligation of candor toward the tribunal (Rule 3.3). Facts: Attorney representing an insurance company was advised by an agent of the insurance company that he had lied in discovery about not receiving the insureds notice. The agent had shredded the letter and altered the mail log to conceal the fact of receipt. Result/Rationale: The Committee determined that in this pretrial situation, the lawyers duty of candor toward the tribunal qualifies her duty to keep client confidences. Continued participation by the lawyer in the matter without rectification or disclosure would assist the client in committing a crime or fraud in violation of Rule 3.3(a)(2). Is perjury in discovery a violation of candor to the court (3.3) of truthfulness to others (4.1)? Candor to the court o Rule 4.1 does not trump Rule 1.6 (duty of confidentiality) o Rule 3.3 does trump Rule 1.6 (duty of confidentiality) Supplement: Jorgenson v. County of Volusia (11th Cir. 1988) Failure of counsel to cite controlling law in seeking temporary restraining order against enforcement of adult entertainment ordinance against bar that featured topless dancers violated Rule 11 and warranted imposition of sanctions. CHAPTER 9 FAIRNESS IN LITIGATION Discussion Problems ABA Model Rules 3.1 through 3.9 and 4.4 1. You represent twelve plaintiffs in an action against Monolith Consolidated Industries, Inc. for race discrimination in employment. The case will be tried before a jury. As is customary in your community, the names and addresses of the 150 citizens on the jury panel have been published in the local newspaper (this is not normal practice). A list of jurors can usually be obtained from the clerk once the names are drawn, this list may also contain information regarding employment With federal court there is no advance list available. But you can identify them when they are impaneled and if your case is not first you may have time to find out something about them. 60 | P a g e

Some lawyers keep files on jurors. Some talk to personnel officers at local plants and find out about potential juror. Some talk to those who know everyone in the community. Juror consultants can be employed. Judges often use lengthy jury questionnaires in high-profile cases that include questions asked by counsel on both sides. a. May you hire a private investigator to find out whatever he can about the attitudes of individual jury panel members toward race discrimination? Juror consultants can be employed Rule 4.4(a) we should not use means that will embarrass, delay, or burden third persons Rule 3.5 cannot seek to influence a juror by a means prohibited by law or communicate ex parte with a juror b. May you have your paralegal assistant search the public records in the County Records Office to find out which jury panel members own real property? Yes public records are available to anyone To find out the political party affiliation of those jury panel members who are registered to vote? Yes c. In the jurisdiction where you plan to practice, do trial lawyers usually obtain information about jury panel members in advance of voir dire? How do they obtain it?

2. During the trial of the above case, you and one of the jurors find yourselves riding the elevator together on the way up to the courtroom one morning. a. Suppose the juror turns to you and says: Well, Good morning, counselor! Do you have some red-hot testimony to keep us all awake today? How should you respond? You should be evasive (softpedal). Dont be rude. Cant talk to them b. Suppose the juror turns to you and says: Well, good morning, counselor! What did you think of that lousy ball game on TV last night? How should you respond? You should be quickly and neutrally as possible Im sorry but Im not permitted to speak to jurors outside of the courtroom. Rule 3.5 (a) and (b) A lawyer shall not: (a) seek to influence a judge, juror, prospective juror or other official by means prohibited by law; (b) communicate ex parte with such a person during the proceeding unless authorized to do so by law or court order; 3. During the noon recess in the above case, you are having lunch at a caf near the courthouse. You observe a female juror sitting at a secluded table in the back, talking in hushed tones with a young man, a law student who works part-time as a paralegal for the firm that represents your adversary. What, if anything, should you do about that? There is no rule about this. First talk to opposing counsel. You cant communicate ex parte with the judge. Then together talk to the judge. Once you talk to the judge it may be out of your hands. You could 61 | P a g e

suggest pulling in an alternate in her place or go forward with one less juror (usually the alternates are not identified until the end, but if she is biased you want her away from the other jurors whether or not she is an alternate). However, you risk the enmity of the other jurors when they learn this juror was removed. Be prepared to deal with this. *8.3(a) * If the attorney told the paralegal to do it 8.4 and 5.3(c) *Common courtesy is to go to the opposing counsel (paralegals boss) first and let him know about the situation, then the 2 of you together may go talk to the judge 4. In the above case, the jury returned a verdict against your clients; judgment was entered accordingly, and the jury was dismissed. a. On your way out of the courthouse, may you stop to chat with one of the jurors, to ask for her comments on the way you presented the evidence and on your closing argument? Yes, generally unless there is some rule or law that prohibits it (or if she is a prospective juror for another of your cases) In the middle and southern districts of Alabama, you must file a motion and get leave of the court to do so (this motion must be served on the other side). You can write a juror a letter thanking a juror for serving on a case after it is over unless local rules prohibit it. Reasons you might want to interview jurors: Where you suspect there is an impaired verdict because it is a quotient verdict (jurors average what all jurors thinks the verdict should be to arrive at a verdict that they agreed beforehand to accept) this is improper and is grounds for a new trial Where you suspect improper influence in the jury Limitations 3.5(c) Cannot ask what anyone say Cant ask about a specific piece of evidence Cant ask about what was said in deliberations b. The day after the trial ended, you learned that during jury deliberations the foreman asked the bailiff for a deposition transcript that had been used at the trial. Part of the transcript had been received in evidence, but the rest had not. Apparently, the bailiff delivered the entire transcript to the jury room. May you interview some of the jurors to find out whether this is true? Yes [since it is not federal court and does not violate Rule 3.5(c)] you could also seek to find this information by asking the court reporter (who has custody of the documents) or the bailiff (but be careful how you ask perhaps ask him to sign an affidavit) Rule 3.5(c) A lawyer shall not: (c) communicate with a juror or prospective juror after discharge of the jury if: (1) the communication is prohibited by law or court 62 | P a g e

order; (2) the juror has made known to the lawyer a desire not to communicate; or (3) the communication involves misrepresentation, coercion, duress or harassment; 5. At the pretrial conference in the above case, you and counsel for Monolith exchanged lists of prospective trial witnesses. Ms list included Erna Tuttle, an investigator employed by the State Fair Employment Practices Commission. Without consulting counsel for M you interviewed T before trial. You learned that she had investigated M and had come away with a favorable impression of the companys hiring practices. Was it proper for you to interview her? There are two types of witnesses expert witnesses (testifies to hypotheticals) and lay/fact witness (testifies to actual knowledge) Attorneys communication with expert witnesses is limited by discovery rules. Fact witnesses are not owned by either side, regardless of whom they may favor. Either side may communicate with fact witnesses as long as they are willing to communicate. Rule 4.2 a lawyer cannot communicate with someone who is represented in the matter without the permission of their counsel Rule 4.3 should not state or imply that we are disinterested, when we are not actually disinterested 6. Prior to trial, and without consulting counsel for M, you contacted several former and current M employees to see if they had information that would support plaintiffs claims. Was that proper? It was okay to contact former employees but the contact of the current employees depends on their level in the organization. Rule 4.2 and comment 7 Rule 4.2 comment 7 In the case of a represented organization, this Rule prohibits communications with a constituent of the organization, who supervises, directs or regularly consults with the organizations lawyer concerning the matter or has authority to obligate the organization with respect to the matter or whose act or omission in connection with the matter may be imputed to the organization for purposes of civil or criminal liability. Consent of the organizations lawyer is not required for communication with a former constituent. 7. One of your trial witnesses in the above case was Edgar Taylor, a former assembly line employee of Monolith. At trial, he testified about a conversation he had overheard between a union official and the head of Ms personnel department. Shortly after he overheard that conversation, M laid him off. He had to move out of state to find a new job. When you asked him to be a witness at the trial, he refused, stating that he did not want to take time away from his new job. To convince him to come, you promised him that your clients would pay:
o o o

His travel, hotel, meals and incidental expenses Was that proper? Yes His lost wages, due to time away from his job Was that proper? Yes One hundred dollars per day, as compensation for his time and trouble in coming to testify. Was that proper? Probably will not be disciplined for this amount, but there is a risk that he will be asked on cross examination whether he is being paid to testify.

63 | P a g e

o o

Was it proper to interview him without notifying Ms counsel? Yes (he is a fact witness and a former employee) Fact witness Can pay out of pocket expenses. Can pay for loss of time for preparation to testify. These amounts must be reasonable.

8. In preparing the above case for trial, you requested production of a large volume of Ms employment records. After the usual preliminary skirmishing, adversary counsel finally agreed to produce them. As is common in such cases, you and adversary counsel orally agreed that M would copy the records (at your clients expense), deliver them to you, and keep the originals available to you should you desire to examine them. The copies M delivered to you (nine large boxes full) were legible, but so light as to be extremely tedious to read. Later, after you had paid the copying bill, you learned (quite by accident) that adversary counsel had instructed the copying machine operator to make the copies hard to read. What should you do in this situation? How you react may impact future relations with this lawyer. If you can handle this matter lightly (make a joke of it), do so. If that doesnt work you can file a Rule 37 motion to compel production. However, you should try to talk to the other attorney first and give him a chance to correct before we file for sanctions. Also confirm this request in writing so you can attach it to your motion (along with a sample of the copies). Rule 4.4(a) In representing a client, a lawyer shall not use means that have no substantial purpose other than to embarrass, delay, or burden a third person, or use methods of obtaining evidence that violate the legal rights of such a person. Rule 3.4 Fairness to Opposing Party and Counsel 9. At the final pretrial conference in the above case, the trial judge heard oral argument on the admissibility of several key items of evidence. The trial judge decided not to rule on the point until after her law clerk could complete his own search of the authorities. Two days later, while reading the newest advance sheets, you found an evidence decision that directly supports your argument. What is the proper way for you to call the new decision to the courts attention? Rule 3.5 this is a matter of local rules; should not communicate ex parte with the judge could call the law clerk and ask how to go about doing this if the clerk cannot tell you, file a motion with the court for leave to submit addition authority 10. The trail of the above case has attracted considerable attention from the local news media. The trial judge has forbidden cameras and recorders inside the courtroom, but during every recess reporters swarm around the trial participants asking questions. Late one afternoon, the trial judge calls the evening recess just after M has completed the direct examination of one of its key witnesses. You gravely doubt the credibility of this witness, and tomorrow morning you will have a chance to cross-examine. As you walk down the courthouse steps, news reporters surround you, asking you to comment on the witnesss testimony and on your plans for tomorrows cross. Considering both ethics and tactics, how will you respond? No comment. It would serve no purpose to tip off the other side. 64 | P a g e

Rule 3.6 Trial Publicity if the statement has a substantial likelihood of prejudicing the case then it is not appropriate (a) prohibits extra-judicial statements that are intended to influence the trier of fact (b) tells what the lawyer may do (innocuous statements that are benign) (c) a lawyer can respond to prejudicial statements if necessary to avoid the prejudice see Comment 7 WITNESS COACHING A lawyer may discuss the case with the witnesses before they testify. If fact, this is part of competence. There is an ethical duty to do so. When a lawyer discusses the case with a witness, the lawyer must not try to bend the witnesss story or put words in the witnesss mouth. What is typical and appropriate: o Discuss the witnesss perception, recollection, and possible testimony about the events in question o Review documents and other tangible items to refresh the witnesss memory or to point out conflicts and inconsistencies with the witnesss story o Reveal other tangible or testimonial evidence to the witness to find out how it affects the witnesss story o Explain how the law applies to the events in question o Review the factual context into which the witnesss testimony will fit o Discuss the role of the witness and effective courtroom demeanor o Discuss probable lines of cross-examination that the witness should be prepared to meet o Rehearse the witnesss testimony, by role playing or other means A lawyer can be disciplined by the bar for counseling or assisting a witness to testify falsely or for knowingly offering testimony that the lawyer knows is false Three grades of witness coaching: o The lawyer knowingly and overtly induces a witness to testify to something the lawyer knows is false o The lawyer knowingly and covertly induces a witness to testify to something the lawyer knows is false o The lawyer does not knowingly induce the witness to testify to something the lawyer knows is false, but the lawyers conversation with the witness nevertheless alters the witnesss story Colorado Bar Association Opinion 70 (1985) After a verdict has been returned, it is improper for an attorney who has participated in the trial to tell the jury about information that was not presented at trial, if such information is disclosed to the jury with the intention of or in the spirit of criticizing the jurys decision, influencing the actions of jurors in future jury service, harassing the jury, or otherwise behaving improperly toward jurors in any manner prohibited by the Code of Professional Responsibility. The practice of talking informally with willing jurors after a trial is a common one in our state courts, although it is not permitted in the federal courts. 65 | P a g e

If an attorney becomes aware of improper communications with a juror by an attorney, ,he has an obligation to make a prompt report to the court regarding such conduct.

PUBLIC COMMENTS ABOUT PENDING LITIGATION Background The right to a public trial is part of the 6th Amendment protection for the criminally accused. Excessive publicity may also interfere with the equally important right to a fair trial. Historically, the 1st Amendment has required a showing of actual prejudice or a substantial and imminent threat to fair trial in order to restrict press coverage during a criminal trial, but has allowed trial courts to restrain lawyers speech before and during trial on a significantly lower showing. The ABA Enters the Fray The Gentile Case In Gentile v. State Bar of Nevada (U.S. 1991) the Supreme Court directly addressed whether state-imposed limitation on extrajudicial statements in criminal cases violated the right of free speech. The Court stated that the regulation of attorneys speech is limited. The Court struck down parts of the Nevada rule which specified that an attorney could make general statements about defense without elaboration. o This rule is confusing o This general versus elaboration directions failed to even consider that a general statement could materially prejudice a case just as much as any elaboration on the defense. The Court upheld the general test but struck down the more explicit language of the rule. Current Rules ABA Model Rules 3.6 and 3.8 were amended to reflect the Gentile decision Matter of Vincenti (NJ 1983), p.223 Shameful display of atrocious deportment calling for substantial discipline Attorney is suspended from the practice of law for one year and until the further order of the court (he would have to reapply for admission) THE PROSECUTORS SPECIAL DUTIES Rule 3.8 Prosecutors have the unique power to bring criminal prosecutions on behalf of the government. The prosecutor is asked to assume a dual role as a partisan advocate and a quasi-judicial officer. 66 | P a g e

The special ethical responsibilities include restraint in prosecuting charges without probable cause; protecting the accuseds right to counsel and other important pretrial rights; disclosing evidence that negates guilt or mitigates the offense or sentence; and exercising restraint in litigation and out-of-court statements. The Due Process Clause requires a prosecutor to disclose evidence that favors the defendant with respect to guilt on the merits, or impeachment of prosecution witnesses, or punishment for the offense. This standard is met if there is a reasonable probability that, had the evidence been disclosed, the result of the proceeding would have been different. Read v. Virginia State Bar- pg 226- prosecutors only had to disclose exculpatory material only when required by law. The Bar recommended that he be disbarred for not disclosing the evidence of the eyewitnesses changed testimony, and therefore was willing to see the defendant convicted of arson, without allowing this eyewitness testimony. Virginia Supreme Court refused to disbar him b/c no harm no foul, the evidence came in, so it didnt really matter. Additional Notes o Prosecutors must make timely disclosure of all evidence that may negate the guilt of the accused, whether or not he believes it o Note the differences between the standards in 3.6 and 3.8

THE DUTIES OF THE CRIMINAL DEFENSE LAWYER The criminal defense lawyer may require the prosecution to put on its proof even if there is no non-frivolous defense. Why I Defend Guilty Clients Supplement Nichols v. Seaboard Coastline Railway (Ala. 1976) Held that one jurors presenting to others, during deliberations, encyclopedia definitions of negligence, contributory negligence, subsequent negligence, and subsequent contributory negligence and four other jurors consulting either that encyclopedia or dictionary to clear up confusion concerning several legal words and phrases constituted prejudice as matter of law (extraneous information) and required reversal. Clarke-Mobile Counties Gas District v. Reeves (Ala. 1993) Held that: (1) one jurors statement to other jurors concerning her experiences with gas company was the equivalent of extraneous facts, and (2) extraneous facts prejudiced the verdict. Gaylard v. Homemakers of Montgomery, Inc. (1996) Lady was burned by attendant while being bathed in a nursing home. The injureds attorney contacted the attendant. 67 | P a g e

Held that (1) clients attorney did not violate Rules of Professional Conduct when he contacted home health care service employee; (2) any violation of Rules of Professional Conduct would not, in any event, justify excluding employees prior inconsistent statements; and (3) erroneous exclusion of prior inconsistent statement was not harmless This conduct does not make the evidence inadmissible, even though it violated the rule of professional ethics. The correct remedy is disciplinary action. PC rules do not govern admissibility of evidence.

Lammon v. Lammon (Ala. Civ. App. 1996) Held that disqualification of attorney based his conduct in making contact with the mother (the other party) while he knew she was represented, which may have violated Rules of Professional Conduct, was not abuse of discretion. A trial court has the discretion to disqualify counsel for violating the rules of professional conduct. A court can disqualify counsel for violation of PC rules. Golden Door Jewelry Creations v. Lloyds Underwriters Non-Marine Association (S.D. Fla. 1994) Held that (1) insurers payment of $753,081.42 to witnesses, informants, and intermediaries did not support violation of statute prohibiting bribery of public officials and witnesses for purposes of determining whether sanctions should be imposed insurer, (2) insurers payment, with knowledge of its counsel, of $120,000 to fact witnesses to testify at depositions in insureds civil actions for insurance monies violated Florida Rule of Professional Conduct prohibiting lawyer from offering inducement to witness that is prohibited by law; and (3) because of ethical violations by insurer and its counsel, sanctions against insurer were warranted. Lay Witness may be interviewed by any attorney Corporate Witnesses may be interviewed under rule 4.2 if Employee has counsel Witness may be paid for time, lost wages etc. for preparing his testimony Expert witness compensation- cant be contingent on Experts testimony, or on the outcome of the case Preparing a Witness to testify-you can certainly talk to the Witness about his perception, memory, review documents and physical evidence, point out inconsistencies, Its ok to have a dry run with what he is going to say, tell him what to wear, tell him why his testimony is important, but.it is NOT ok to tell the Witness what to say CHAPTER 10 BIAS IN (AND OUT OF THE COURTROOM) Discussion Problems:

68 | P a g e

1. Dan Mitchell is a partner in a mid-size law firm. He has assigned Beth Hammond, one of the firms best associates, to a litigation matter for his client, Modern Furnishings. Until now, Beths work has involved research and drafting pleadings, but an important deposition is scheduled in three weeks. During a meeting between Dan and George Drake, the president of MF, George suggests that Beth might not be the best choice for taking the deposition. When pressed George states that although Beths work has been excellent, he would be more comfortable having a male associate conduct the deposition due to its importance to the litigation and the necessity for an aggressive approach. Does Georges request reflect bias? Yes What should Dan do? What should Beth do if she learns of Georges concerns? Be sensitive to Beth, but you also have to keep the client happy if you want to keep the client. 2. Ethan Baird, age 73, was involved in an automobile accident injuring Cathy Stevens. Cathy sued Ethan to recover for her injuries; whether Ethan was negligent was disputed. While cross-examining Ethan, Cathys attorney asked repeated questions concerning Ethans age, his vision, and the length of time since his last driving test. At the conclusion of his cross-examination, Cathys attorney commented, The State needs to do a better job of keeping old coots off the highway. Does Cathys attorneys questioning reflect bias? No. Is Cathys attorney questioning appropriate? These are all relevant questions. What about her comments? Inappropriate from a practical point of view it would probably have a negative impact on the jury 3. The deputy district attorney for the City of Brownstone was faced with the decision whether to indict a Mexican-American for an alleged assault. Responding to a colleagues question as to whether he intended to proceed with an indictment, the deputy said yes. The colleague noted the conflicting evidence in the case, and questioned whether the deputy had any hesitation. The deputy replied, Those people always have an alibi-they stick together. Does the deputy district attorneys reply bias? Yes Does it raise any ethical concerns? Yes, however, no sanctions 4. Jennifer Holden is a twenty-eight year-old associate attorney at a large law firm. Jennifer works with Sam Baker, a partner in his mid-fifties. Clients regularly mistake Jennifer for a secretary, paralegal, or court reporter instead of a lawyer. Does this mistaken identity reflect bias? Probably not Why would this mistaken identity occur? What steps could be taken to avoid such misapprehensions in the future? Sam should take the lead and be aggressive about introducing Jennifer to people as an attorney. 5. Under 2007 new rule, he has to resign under rule 3.6. The old rule says that he wouldnt have to resign. Federal Judges are prohibited from joining any groups like this. Theyd have to resign before nomination hearings. b. Rule 2.3- judges cant harass etc. based on race, gender etc. and shall not permit court staff from doing so. Hed be subject to discipline for this 69 | P a g e

c. Yes, b/c he has violated rule 2.3 also because the bailiff is under his control at this time, and so the judge has a duty to deal with it at the time. d.

The Study of Gender in the Courts: Keeping Bias at Bay (2002), p.237 A lack of apparent intention behind documented misbehavior does not mean there is no impact. Whats Gender Got To Do With It? Incivility In The Federal Courts, p.237 Study in the Eighth Circuit found that almost two-thirds of attorneys have experienced general incivility, gendered incivility, and/or unwanted sexual attention while in litigation. Women encountered such mistreatment more frequently than men. In Re Charges of Unprofessional Conduct Contained in Panel File 98-26 (1999), p.238 Facts: An African-American male was charged with felony robbery of a Caucasian couple. The defendants counsel believed race was a motivating factor in the indictment and intended to have an African-American join as cocounsel in the case. The prosecutor filed a motion seeking a court order which would prohibit counsel for the defendant to have a person of color as co-counsel for the sole purpose of playing upon the emotions of the jury. The prosecutor was charged with unprofessional conduct and was given a private admonition (which is punishment only for non-serious offenses). Result: This was serious offense and merely a private admonition was not appropriate. Rationale: o A counsels race should never be used as a basis for calling for or placing limits on that counsels participation in any court proceedings. o The motion in this case could be reasonably interpreted as implying that an attorney of color would have no legitimate use on the case other than to arouse the passions of the jury. o The motion was precariously close to infringing on defendants right to be represented by counsel of his choice. In Re Plaza Hotel Corporation (Bankruptcy Court 1990), p.240 Facts: During this bankruptcy case the debtors counsel referred to counsel for the U.S. Trustee as office help even though he knew she was a lawyer. He also had a condescending attitude towards her during the hearing. Result: Disqualified the debtors counsel In Re Vincenti (N.J. 1998), p.240 Facts: An attorney harassed and intimidated a social worker on the basis of sexual orientation. 70 | P a g e

Result: Disbarred

In Re Kirby (Minn. 1984), p.242 Facts: The judge in several instances was publicly intoxicated, was conducting judicial business with liquor on his breath, had a pattern of habitual tardiness, and was discourteous to women lawyers, including two occasions in which he referred to female attorneys as lawyerette and attorney generalette. Result: Public censure Rule: o Cannon 3A(3) of the Code of Judicial Conduct A judge should be patient, dignified, and courteous to litigants, jurors, witnesses, lawyers and others with whom he deals in his official capacity. Still Officers of the Court: Why the First Amendment is no Bar to Challenging Racism, Sexism and Ethnic Bias in the Legal Profession, p.244 Task force reports on racial, ethnic and gender bias in the administration of justice have consistently found evidence of a wide range of discriminatory conduct by lawyers directed toward witnesses, other lawyers and court personnel. CHAPTER 11 CONFLICTS OF INTEREST LAWYERS, CLIENTS, AND THIRD PARTIES Discussion Problems Rules 1.2, 1.7, 1.8, 1.10, 1.13, 1.14, 1.18, 2.1, 3.7, and 5.4(c) 1. Attorney Whartons law practice consists of insurance defense work. Hamilton Casualty Co. has hired her to defend Silas Combs in a negligence case. The plaintiff in the case alleges that Combss rice field was plowed negligently, so as to cause a large quantity of water to escape into plaintiffs adjoining tomato field. The water caused plaintiffs tomatoes to rot before harvest. Ps complaint demands $125,000 in damages. Combs insurance policy with Hamilton Casualty has a top liability limit of $100,000. After extensive discovery, the case was set for trial. Six weeks before the trial date, plaintiffs lawyer called Wharton and offered to settle the case for $90,000. What are Whartons ethical obligations in this situation? Must inform his client of the settlement offer. Whether to accept the offer is the decision of the client (the insured). Rule 1.2(a) dual representation of the insurer and the insured: split of authority as to whether one can represent both primary responsibility is clearly to the insured there is potential for conflict here Hamilton may not want to settle and may believe it is more advantageous to them to go to trial since this amount is so close to their top liability limit anyway. Combs will want to settle because he will not be out anything. Attorney Wharton may not want to take a position contrary to Hamilton because she will want to continue getting business from them Wharton should tell Combs that he has a right to make a demand on the insurance company to settle for the $90,000 Wharton may withdraw if he feels 71 | P a g e

there is too big of a conflict to make the demand on the insurance company for Combs himself Rule 1.8(f) & Rule 5.4(c) goal is to avoid third party interference Rule 1.7 Any conflicts must be resolved in favor of the insured! 1.0(e) definition of informed consent 118(f) & 5.4(c) problems

2. Consider the following transcript that depicts a meeting between Glenn Sparks (the president of a large electric appliance manufacturing corporation) and Helen Wilson (the in-house general counsel for the corporation). Ms. Wilson and Mr. Sparks discuss three legal problems that face the corporation. The first problem concerns a new stock issue. Ms. Wilson has discovered a potentially embarrassing contingent liability that should be disclosed in the SEC registration statement. Mr. Sparks foresees practical problems if it is disclosed, and he attempts to resolve the issue by relieving Ms. Wilson of her duties in connection with the stock issue. The second problem concerns a products liability suit in which it appears that the corporations interests might be served by delay and by putting the plaintiff through expensive discovery proceedings. The third problem is related to the second. The plaintiffs own discovery in the products liability suit may uncover some highly questionable conduct by the corporation involving secret payments to labor union inspectors. Ms. Wilson has repeatedly advised against this conduct, but to no avail. As you read the transcript, consider these questions: Rule 1.13
o

Who is Ms. Wilsons client here? The corporation Rule 1.13(a) a lawyer retained by an organization represents the organization the individuals are not the client Has Mr. Sparks asked Ms. Wilson to do anything that you regard as clearly illegal or clearly unethical? Rule 1.13(b) her duty is to the organization if she knows an officer plans to violate the law in a way that will cause injury or cause liability to the organization here the president is a third party attempting to interfere in the attorney-client relationship she should act reasonably she might ask for reconsideration of the matter, might suggest separate legal opinions be sought, might refer the matter to higher authority in the organization She should counsel them to obey the law and she should not assist in fraud Sparks is not her responsibility. Sarbanes Oxley implicated and changed this rule to the present form

72 | P a g e

o o

o o

If so, has Ms. Wilson sufficiently insulated herself from the illegal or unethical conduct? No her duty to the organization is not related to the work she is doing for the organization but to the information she has that is detrimental to the organization she may withdraw it need be even if she allows outside counsel to handle it she is off the hook because she is not assisting, but this does not get him off the hook as to the organization because she has knowledge she has a duty to inform the organization (her client) Has she effectively served her client? No 3.4(d) lawyer should not make a frivolous discovery request In discovery they will turn up criminal and civil liability and this knowledge giver her a duty to inform the organization There is a bribe here if we can work this out you will be well paid Conflicts between employees and the organization can arise rule 1.13(d) says that the lawyer must explain to an employee that she represents the organization and cannot represent the employee if his interests are in conflict if the employee gives confidential information to the lawyer in anticipation of being represented, then the lawyer is out altogether and cannot represent either the organization or the employee in that matter can represent an employee if the interest is not in conflict with the organization

For transcript, see pages 251-253 of casebook. 3. The Sarbanes-Oxley Act of 2002 was signed into law on July 30, 2002. The Act directs the Securities and Exchange Commission to adopt rules of professional responsibility requiring attorneys to report a material violation of breach of fiduciary duty or of securities laws to the issuers chief legal officer (general counsel) or chief executive officer. If there is no adequate response to the report, attorneys would be required to report such material violations to the audit committee or to the board of directors. How would such a provision affect the scenario in In re Electro Corp., above? Would require the attorney to report Rule 1.13(c) [there is a new rule] gives the lawyer some discretionary power to report, instead of resign this was adopted in 2003 and will not be on the MPRE until 2004 1.13(c)(2) trumps 6.2

4. After attorney Sarah graduated from law school, she opened her own law office in a small seaside village. She longs for a cottage on the beach, but she has been unable to find one at the right price. Client Willis has retained her to help him find a way out of his financial distress. Among his few solid assets is a lovely cottage on a secluded end of the beach. Willis has been unable to pay the taxes on the cottage, and Sarah has advised him to put it up for public auction.

73 | P a g e

a. At the auction, may Sarah have her brother bid for her as undisclosed principal? No this would be a conflict and improper Rule 1.8(b) A lawyer shall not use information relating to representation of a client to the disadvantage of the client unless the client gives informed consent, except as permitted or required by these Rules Rule 1.7(a) Except as provided in paragraph (b), a lawyer shall not represent a client if the representation involves a concurrent conflict of interest. A concurrent conflict of interests exists if: (2) there is a significant risk that the representation of one or more clients will be materially limited by the lawyers responsibilities to another client, a former client or a third person or by a personal interest of the lawyer b. Suppose, instead, that Sarah simply agrees to buy the cottage directly from Willis, subject to the tax debt. Under what, if any, circumstances would that be proper? Would be proper only if (1) the terms are fair and reasonable and fully disclosed in writing (2) the client is advised in writing and given a reasonable opportunity to seek the advice of independent counsel (3) the client gives informed consent in writing to the essential terms and the attorneys role in the transaction See Rule 1.8(a)- 4 requirements set forth in the rule- listed above She should let Willis select an appraiser and consult with independent counsel. c. Suppose, instead, that Sarah agrees to lend Willis enough money to pay off the back taxes on his cottage. Under what, if any circumstances would that be proper? Not proper Rule 1.8(e) cannot provide financial assistance to a client in connection with pending or contemplated litigation (but this is not pending or contemplated litigation) would have to comply with the requirements of Rule 1.8(a) Not she has 1.7(a)(2) conflict of interest by being his lawyer and his creditor, so must comply with 1.7(a) 5. Jefferson and Herchberger are involved in a boundary line suit concerning twelve acres of land that lies in a valley between their two farms. Lawyer Lennihan represents Herchberger in the suit. a. May Lennihan purchase from Jefferson a 30% interest in that twelve acres? No Rule 1.8 this would create an interest directly adverse to his client and 1.7 conflict of interest problem. b. May Lennihan purchase from Herchberger a 30% interest in that twelve acres? Rule 1.8(i) A lawyer shall not acquire a proprietary interest in the cause of action or subject matter of litigation the 74 | P a g e

lawyer is conducting for a client could lead to a conflict between the lawyer and client with settlement offers c. May Lennihan agree with Herchberger to do the legal work in exchange for a 30% interest in that twelve acres if Herchberger wins the suit? Yes Rule 1.8(i)(1) exception for contingent fee contract 6. Client Curt hired attorney Annette to advise him during some difficult business negotiations with Danforth Corporation. The negotiations extended over many months; during that time, Annette developed a good working relationship with Curt and a thorough understanding of the factual and legal problems at hand. Only four people were present at the negotiating sessions: Curt, Annette, Danforths vicepresident, and Danforths house counsel. Ultimately the negotiations failed, and Danforth sued Curt. A key contested issue at trial will be whether Curt made a certain statement during one of the negotiating sessions. Curt wants Annette to represent him at trial, but Danforth has moved to disqualify her on the ground that she may have to testify about Curt making the alleged statement. a. Should the court grant the motion to disqualify? Yes, unless there is a substantial hardship on the client Rule 3.7(a) A lawyer shall not act as advocate at a trial in which the lawyer is likely to be a necessary witness unless: (1) the testimony relates to an uncontested issue; (2) the testimony relates to the nature and value of legal services rendered in the case; or (3) disqualification of the lawyer would work substantial hardship on the client b. May Annettes law partner, Elmwood, serve as Curts trial lawyer? No Rule 3.7(b) A lawyer may act as advocate in a trial in which another lawyer in the lawyers firm is likely to be called as a witness unless precluded by Rule 1.7 [conflict of interest] or Rule 1.9 [conflict between client and a former client] this is an exception to imputed conflicts of interest / imputed disqualification Rule 1.10 (which only applies to Rule 1.7, 1.8(k), 1.9) However, if her testimony will be adverse to Curt, she has a conflict of interest. Rule 1.7 doesnt list every conflict of interest, but says that an attorney shall not represent a client if the representation involves a concurrent conflict of interest. c. Does your answer to either question depend on whether Annettes testimony would be for Curt or against Curt? YES 7. For many years, attorney Alice has looked after the legal and financial affairs of her client Chadbourne, an aged widower. Chadbourne has asked Alice to prepare a new will for him, but he does not know whom to name as executor. He does not want an institutional executor, and he has no suitable friends or relatives. a. May Alice suggest herself as executrix? YES See Rule 1.8(c) cannot solicit a gift; Rule 1.8, Comment 8 may suggest herself, subject to general conflict of interest provision in Rule 1.7 75 | P a g e

must obtain informed consent to the conflict should not overreach, should not lean on the client (duress), should offer options and let the client make the decision b. May Alice accept as thanks for all her kindness to Chadbourne over the years a modest picture frame she has admired when visiting Chadbournes townhome? Yes Rule 1.8, Comment 6 c. May Alice accept Chadbournes valuable townhome if given to her as a gift? What if it is left to her in Cs will? NO Unless another lawyer is involved. Alice cannot prepare the legal document, and because of imputed disqualification neither can any other lawyer in Alices firm. Another lawyer must provide advice and prepare the will or deed this would not be considered a close familiar relationship Rule 1.8(c) and Comment 7 8. The law firm of Shubert, DeWitt and Howe specializes in family law matters. The firm has three partners and four associate lawyers. a. Partner Rhonda Howe is representing client Curt Clinton in a pending dissolution of marriage proceeding. Howes own marriage was recently dissolved, so she is especially sympathetic to Clintons situation. On several occasions, she and Clinton have discussed Clintons legal problems over long dinners. Howe now finds herself quite attracted to Clinton as a person, and she believes the attraction is mutual. What advice would you give Howe in this situation? Advise in the context of personal interest conflict. This could cloud her judgment or make him be less than candid with her. She should not have any sexual relationship with the client until the case is over. Rule 1.8(j) and Comments 17, 18, 19 (doesnt apply to imputed disqualification of 1.8(k) and theres an excpetion when relationship starts before he becomes a client. Rule 1.8(k) imputed disqualification does not apply in these relationship situations b. Partner Shubert has been asked to represent client Cummings in a child custody dispute with Cummings ex-husband. Attorney Arnott, a partner in a different law firm, represents the ex. Arnott and Shubert are engaged to be married in the near future. Shubert believes that Cummings matter could be handled adequately by the senior associate in Shuberts firm. What advice would you give Shubert in this situation? Allow the senior associate to handle the matter there is no rule dealing with this specific problem, but this would be a personal interest conflict imputed disqualification ???does it apply here??? 1.7 Conflict of Interest disqualification--- hes disqualified b/c this is a 1.7 conflict

76 | P a g e

1.10(a) Disqualification is imputed to other members of a firm if arising under Rule 1.7 and rule 1.9- unless prohibition is based on personal interest. . .

Three types of conflicts: Conflicts with third parties Conflicts between the attorney and client Conflicts between two clients or a client and a former client Allocating Decision-Making Between Lawyer and Client Routine and technical matters, including those ordinary matters which arise in the course of litigation, may be handled independently by the attorney as a necessary aspect of the professional management of the case. Decisions which affect substantial rights, whether procedural or substantive, must involve the client. o Substantial rights means important or essential rights, rights affecting the merits of the cause or serious steps in the litigation The client defines the goals of the representation and the lawyer implements them, but each consults with the other. Beckwith Machinery Co. v. Travelers Indem. Co. (W.D. Penn. 1986), p.257 An insurer has a dual duty duty to indemnify and duty to defend Result: o The Trumbull complaint stated claims that were potentially covered by the insurance policy issued by Travelers to Beckwith o Travelers breached its duty to defend Beckwith in the underlying Trumbull case o Travelers failed to reserve its rights to contest indemnity regarding the compensatory damage claims raised in the Trumbull case o Travelers is estopeed from denying coverage because Beckwith detrimentally relied on Travelers policy for indemnification Ethics in the Practice of Law Identifying the client is critical for the lawyer retained by an organization, identification of the client can be complicated The lawyer may need to give some sort of Miranda warning to those officers the lawyer treats as nonclients. Phillips v. Carson (Kan. 1987), p.263 Facts: There was an attorney client relationship between Attorney Carson and Mrs. Phillips. Carson secured loans from Mrs. Phillips, advised her or failed to advise her about them, and agreed to take care of preparing and filing all necessary documents in connection. Carson had a duty to properly, competently, and adequately counsel, advise, and represent the client. Carson breached that duty by failing to file the final mortgage of record, in failing to advise her of the legal consequences of the changes in security, in 77 | P a g e

failing to recommend that she secure independent counsel, and in other ways. These breaches cause damages to Mrs. Phillips. Rule: A lawyer shall not enter into a business transaction with a client if they have differing interest therein and if the client expects the lawyer to exercise his professional judgment therein for the protection of the client, unless the client has consented after full disclosure.

AAA Plumbing Pottery Corp. v. St. Paul Ins. Co. of Illinois (N.D. Ill. 1995), p.267 Granted motion to disqualify plaintiffs attorney from representing plaintiff in this case because the attorney is a necessary witness whose testimony may be prejudicial to his client Supplement: L & S Roofing Supply Co., Inc. v. St. Paul Fire & Marine Ins. Co. (Ala. 1988) Insurers decision to defend insured under reservation of rights does not create such conflict of interest that insured is entitled at outset to engage defense counsel of its choice at insurers expense, unless insurer or defense counsel retained by insurer to represent insured fail to meet enhanced obligation of good faith. Such good faith obligation requires through investigation of claim against insured, fully informing insured of reservation-of-rights defense and all developments relevant to policy coverage and progress of lawsuit, understanding that counsel represents only insured and not insurer, allowing insured to make ultimate choice regarding settlement, not pursuing course that is advantageous to insurer and disadvantageous to insured, and closely scrutinizing defense in situation in which outcome of trial would determine whether coverage exists. U.S. v. Hearst (9th Cir. 1980) Defendant filed a motion for order vacating, setting aside or correcting her sentence for armed bank robbery or, in the alternative, for order reducing her sentence. The court held that: o The case was not rendered moot even though the President commuted petitioners sentence o Petitioner was entitled to hearing on issues of whether petitioners 6th Amendment right of assistance was violated when her counsel pursued his own interest in publication rights, rather than her interest in acquittal, by failing to seek a continuance, failing to seek a change of venue, and putting defendant on witness stand o District Court correctly found that petitioner had no possibility of demonstrating that her attorneys representation was incompetent to the extent that he failed to investigate fully the possibility that involuntary ingestion of hallucinogens overcame her will 78 | P a g e

o Petitioners counsels representation in attempting to suppress tape of defendants incriminating statements did not fall below reasonably competent and effective level o Issues as to whether the incriminating tape was fruit of a 4th Amendment violation could not be raised on collateral review o Petitioners claim that prejudicial pretrial publicity made it impossible for her to receive a fair trial was waived by counsels failure to make the claim at trial or on direct appeal Rule 5.6(b) A lawyer shall not participate in offering or making: (a) a partnership, operating, employment, or other similar type of agreement that restricts the right of a lawyer to practice after termination of the relationship, except an agreement concerning benefits upon retirement; or (b) an agreement in which a restriction on the lawyers right to practice is part of the settlement of a client controversy. So, if this is violated, you have a duty to report this under Rule 8.3 CHAPTER 12 CONFLICTS OF INTEREST CONFLICTS BETWEEN TWO CLIENTS Discussion Problems ABA Model Rules 1.7, 1.8(b), (g), and (i), 1.9 through 1.12 and 6.3 1. The County Association of Real Estate Dealers (CARED) is a trade association composed of all the licensed real estate brokers and dealers in the county. Over the years, CARED has provided its members with standard forms of legal documents for use in routine real estate transactions. Last year, the state passed a new statute that requires all legal documents affecting consumers to be expressed in clear, simple English. Any document that does not comply is void. CARED hired attorney Adler to draft a new standard form apartment lease to comply with the new state statute. Adler did so, and CARED distributed the new form to its members. Dearbourne Realty & Investment Company (a CARED member) used the form to lease one of its own apartment units to Leon Beckner. Now Beckner seeks to have attorney Adler represent him in a lawsuit against Dearbourne to have the lease declared void. One of Beckners several arguments is that two key paragraphs of the lease are totally incomprehensible to the average person. Adler has re-examined the two paragraphs and is inclined to agree that Beckner is very likely correct. May Adler represent Beckner in the lawsuit, seeking to void this lease? NO [1.7 if the association is a current client 1.9 if the association is a former client] Rule 1.9 Duties to Former Clients (a) A lawyer who has formerly represented a client in a matter shall not thereafter represent another person in the same or a substantially related matter in which that persons interests are materially adverse to the interests of the former client unless the former client gives informed consent, confirmed in writing. 79 | P a g e

The confidential information trumps May Adler represent Dearbourne? It depends on whether Adler has received any confidential information from Beckner. o Rule 1.18 Duties to Prospective Client o Comment 4 a lawyer considering whether or not to undertake a new matter should limit the initial interview to only such information as reasonably appears necessary for that purpose determine whether or not you have conflict of interest Ask very specific questions o Purpose of this rule is to encourage prospective clients to be candid Issue of positional conflicts Rule 1.7, Comment 24 Ordinary a lawyer may take inconsistent legal positions in different tribunals at different times on behalf of different clients. The mere fact that advocating a legal position on behalf of one client might create precedent adverse to the interests of a client represented by the lawyer in an unrelated matter does not create a conflict of interest. A conflict of interest exists, however, if there is a significant risk that a lawyers action on behalf of one client will materially limit the lawyers effectiveness in representing another client in a different case; for example, when a decision favoring one client will create a precedent likely to seriously weaken the position taken on behalf of the other client. Factors relevant in Consent. determining whether the clients need to be advised of the risk include: where the cases are pending, whether the issue is substantive or procedural, the temporal relationship between the matters, the significance of the issue to the immediate and long-term interests of the clients involved and the clients reasonable expectations in retaining the lawyer. If there is significant risk of material limitation, then absent informed consent of the affected clients, the lawyer must refuse one of the representations or withdraw from one or both matters. 2. Aaron, Bropovski, and Carter (ABC) were riding in a car driven by Duffy. The car was hit, head-on, by a truck driven by Emerson. ABC and Duffy have asked you to represent them in a suit against Emerson and his employer, United Fat and Tallow, Inc. Under what circumstances may you represent the plaintiffs in this case? Rule 1.7(b) (1) the lawyer reasonably believes that the lawyer will be able to provide competent and diligent representation to each affected client; (2) the representation is not prohibited by law; (3) the representation does not involve the assertion of a claim by one client against another client represented by the lawyer in the same litigation or other proceeding before a tribunal; and (4) each affected client gives informed consent, confirmed in writing. Consent by all of the parties A, B, and C probably ok, but D might be problematic (think contrib. negligence) Rule 1.8(g) cant settle multiple claims at once unless the client consents and all parties agree 80 | P a g e

In Alabama - A driver in not liable to a passenger for negligence, only for wanton or willful behavior, unless the passenger is paying the driver, then the passenger can sue the driver for mere negligence. You dont know yet how the facts may develop in this case. If it turns out A,B,&C have a cause of action against D you cannot drop D as a client and sue D. You also cannot defend D against A,B,&C. You cant split the cause of action. You will be out of the case altogether. MUST WITHDRAW ALTOGETHER Up front, do you tell the A,B,&C that they may have a cause of action against D? See State Farm Mutual Automobile Ins. Co. v. K.A.W. (Fla. 1991) below You can represent A,B,&C together. Tell D he needs to get another lawyer. You can refer him to another lawyer. The plaintiffs lawyers can work together as long as no conflicts develop. This gives all the plaintiffs a tactical advantage. Rule 1.8(g) A lawyer who represents two or more clients shall not participate in making an aggregate settlement of the claims of or against the clients, or in a criminal case an aggregate agreement as to guilty or nolo contendere pleas, unless each client gives informed consent, in a writing signed by the client. The lawyers disclosure shall include the existence and nature of all the claims or pleas involved and of the participation of each person in the settlement. If the defense offers an aggregate settlement, you must call all your clients together. If they cannot agree on how to split it, you must turn down the settlement and ask the defense to discuss individual settlements for each of the defendants. 3. You are one of only five lawyers in the little town of Sand Springs. You represent Sand Springs Hardware Company in a civil suit against Virgil McQuillan to collect $338 on an overdue charge account. Last week, McQuillan and one of his friends got drunk and were arrested trying to climb into the back window of a saloon after closing hours. They were charged with burglary, a felony. Neither man has funds to pay a lawyer. a. McQuillans Friend is being represented by the County Public Defender, but she has declined to represent McQuillan too, on the grounds that to do so would pose a conflict of interest. Is her position sound? YES under these circumstances if she represents both she runs into a serious conflict if the prosecution wants to offer a deal to one if they will testify against the other 1.7 Comment 23 page 29 b. The court has asked you to defend McQuillan in the felony case. How should you respond to the courts request? This could create a conflict even though the matters are different, McQuillan may confide information relevant in case two that is also relevant in case one i. See Rule 1.7, Comment 6 4. Two years ago, you represented Mr. W in setting up a close corporation for his business and for certain personal investments. That work has long since been 81 | P a g e

completed and you have not represented Mr. W since then. Now Mrs. W has asked you to represent her in divorce proceedings against Mr. W. The two of them are in sharp disagreement over the division of property, child support obligations, and alimony. Assume that this jurisdiction does not have community property. Under what, if any, circumstances may you represent Mrs. W? Ability to represent depends on whether any information from the first matter will relate to the issues in the second matter in this case there could easily be a conflict because both cases involve Mr. Ws investments See Rule 1.9 Mr. W can represent her against him only with his consent you could ask him for his consent, but Brewer would pass on this one 5. From 1991 through 1999, lawyer Lageson was an attorney with the Argos & Bakesons firm. Argos & Bakesons practice was primarily insurance defense work. While at Argos & Bakeson, L worked on numerous matters for the Wazoo Insurance Company. In 1999, Lageson left Argos & Bakeson for the Cumis form where he also did insurance defense work, including matters in which Wazoo insureds needed separate counsel because Wazoo reserved its rights to contest coverage of the claim. Cumis represents Duke Development, a real estate developer being sued by buyers of several hundreds Dule-built homes for alleged construction defects. Duke has various insurance policies with Wazoo that it contends cover the defect claims. Wazoo disputes coverage and has sued Duke for a declaration that it is not liable for the buyers claims. Argos & B represent Wazoo and Cumis represents Duke in the 1.9(b) declaratory relief action. Wazoo has moved to disqualify Cumis from the 1.10(a)(2) representation based on Lagesons former association with A & B. 1.10(b) a. Should L personally be disqualified from representing Duke in Wazoo Insurance Company v. Duke? YES Wazoo was a former client and seems to be the same or a substantially related matter Rule 1.9(a) b. If Lageson is personally disqualified, should the entire Cumis firm also be disqualified? YES Rule 1.10(a) the disqualification of L is imputed to all the members of the firm (if it is a disqualification through 1.7 or 1.9) (1.8 disqualifications are also imputed except for 1.8(j)) c. Are there any steps the Cumis firm could have taken to avoid its imputed disqualification? Examine as best you can the solutions under the ABA Model Rules and the approach of the Restatement. NO not under the ABA Model Rules See Haagen-Dazs Co. Inc. v. Perche No! Gelato, Inc. and Double Rainbow Gourmet Ice Creams, Inc. (N.D. Cal. 1986) below Note that the rule for government employees and judges allows the Chinese Wall See Roberts v. Hutchins (Ala. 1990) below d. If you believe sufficient screening procedures can protect a private firm from disqualification, can you see why the former client who is the firms adversary still might feel uneasy? 6. Until ten months ago, attorney B was an Administrative Law Judge for the State Consumer Protection Commission. The Commissions Enforcement 82 | P a g e

Division brought proceedings against Mandel Toy rifles. The Commission attorney moved for a preliminary cease and desist order. B was assigned to hear the motion. She declined to issue the order, stating on the record the evidence of dangerousness looks exceedingly thin at this time. Several months later, B resigned her position and entered the private practice of law. Ultimately, after a full hearing before a different ALJ, Mandel was ordered to take the toy rifles off the market. That order is now before the Appellate Division of the Commission, and Mandel has asked B to argue the appeal on its behalf. May she do so? NO Rule 1.12(a) Except as stated in paragraph (d), a lawyer shall not represent anyone in connection with a matter in which the lawyer participated personally and substantially as a judge or other adjudicative officer or law clerk to such a person or as an arbitrator, mediator or other third-party neutral, unless all parties to the proceeding give informed consent confirmed in writing a. Her participation was personal (she entered the order) and substantial (it was not merely an administrative task, she dealt with the merits of the case) b. Her firm may participate if she is timely screened from participation and receives no part of the fee See Rule 1.12(c) i. This fee limitation is of no real impact because most firms pay out of the general common fund and she may share equally in this see comment 4 ii. Written notice must be given to the parties and appropriate tribunal to enable them to ascertain compliance See Rule 1.12(c)(2) iii. This applies to law clerks working for a judge too must inform the judge in the matter before discussing employment with a firm involved in the case Rule 1.11 government officers and employers proscribes representation in matter that person participated in personally and substantially unless there is consent however, the individuals firm can represent if there is timely screening, no apportionment of fees, and notice is given to the parties and appropriate tribunal (same as rule for judges- 1.12(c) o Rule 1.11(c) if the former government officer or employee has confidential government information he cannot represent a private client whose interests are adverse to the person in the confidential matter same screening applies here Rule 1.8(k) Rule 1.9 requires that the matters be related Rule 1.9(c) if you have confidential infoyou will be disqualified Rule 1.10(a)- comment on Pg 39 applies to nonlawyers- paralegal doesnt disqualify a firm, where a lawyer does, so long as she has been screened etc.

State Farm Mutual Automobile Ins. Co. v. K.A.W. (Fla. 1991) p.279 83 | P a g e

Facts: Following a car accident, the Wilkerson family retained the Schlesinger firm and file suit against the driver and owner of the other car, State Farm (for uninsured motorist coverage), and a medical malpractice action against various health care providers. After the Wilkersons attorneys determined that Mr. Wilkersons negligence may have contributed to the accident, Mr. Wilkerson discharged the Schlesinger firm as his counsel in the personal injury action and retained a former member of the firm as new counsel. Mrs. Wilkerson and the daughter (still represented by the Schlesinger firm) filed an amended complaint adding Mr. Wilkerson as a defendant and he consented to be sued up to the limits of his insurance coverage. The Schlesinger firm continued to represent all three Wilkersons in the medical malpractice action. o The defendant insurance companies filed motions seeking disqualification of the Schlesinger firm in the personal injury action because they objected to the potential for the Schlesinger firm to use confidential information gained during the course of the prior representation of Mr. Wilkerson. Result: The Schlesinger firm is disqualified from further representation of Mrs. Wilkerson and her daughter in this action. Rationale: o There are two purposes to the requirement that an attorney maintain client confidences: It advances the interest of the client by encouraging trust and the free flow of information Fundamental to a fair adversary system o Whether insurers may stand in the shoes of their insured for purposes of seeking disqualification of the Schlesinger firm on grounds of conflict of interest YES where a conflict is such as clearly to call in question the fair or efficient administration of justice, opposing counsel may properly raise the question the facts of this case call into question the fair administration of justice Mr. Wilkerson is adverse to the position of his wife and daughter in theory only Information Mr. Wilkerson disclosed to his attorneys during the course of the relationship could be used to prove that he was negligent and this is adverse to the insurance companies who are obligated to act in his defense o One seeking to disqualify opposing counsel must show that an attorneyclient relationship existed (gives rise to an irrefutable presumption that confidences were disclosed during the relationship) and that the matter in which the firm subsequently represented the interest adverse to the former client was the same or substantially related to the matter in which is represented the former client

Haagen-Dazs Co. Inc. v. Perche No! Gelato, Inc. and Double Rainbow Gourmet Ice Creams, Inc. (N.D. Cal. 1986) p.284 Facts: Double Rainbow filed suit alleging that Haagen-Dazs/Pillsbury violated antitrust laws. Haagen-Dazs/Pillsbury filed a motion to disqualify Double 84 | P a g e

Rainbows counsel the Gray firm and the Alioto firm. Mr. Jesse, an attorney formerly employed as in-house counsel for Pillsbury, is now associated with the Gray firm . Result: o Mr. Jess must be disqualified o The Gray firm must be disqualified o The Alioto firm is not disqualified Rationale: o Mr. Jesse worked on matters substantially related to this litigation and there is a reasonable basis for concluding that he received confidential information relevant to this litigation o The Gray firm did not institute the Chinese Wall measures until after the first of these suits had been filed o Mr. Jesses did not have direct contact with the Alioto firm and the association of the Gray firm and the Alioto firm does not warrant disqualification of the Alioto firm. The Alioto firm is required to return to the Gray firm all the documents and information which it ahd obtained from the Gray firm (to purge the tainted information).

JOINT REPRESENTATION Three basic policy considerations that under lie the conflict of interest rules dealing with joint representation: o The interest of clients in certain objectives that are available through joint representation o The need to protect clients from the dangers of joint representation o The desire to preserve lawyers reputations by avoiding apparent impropriety Current rules require the attorney to ascertain whether the conflict between two clients is consentable, and if so, to obtain informed written consent from both clients. IMPUTED DISQUALIFICATION AND SCREENING There is an assumption that lawyers working together in a single firm share each others, and their clients, secrets and confidences as a consequence, lawyers within a firm must share each others disqualifications for conflicts of interest Spousal and other family conflict and the lawyer witness rule do not come within the automatic disqualifications There is no automatic disqualification for all lawyers with whom a departing lawyer has been associated. Most imputed disqualifications can be cured by informed client consent. ethical walls / Chinese walls approval by the courts has been mixed and largely limited to former public officers, government employees, and judges moving to private practice the Model Rules do not formally accept the device in private firm to private firm moves Rule 1.11 when lawyers move from government job to private practice 85 | P a g e

Comment 6- page 42- doesnt prohibit lawyer from receiving a salary or______ but cant get direct compensation ______

Supplement: Roberts v. Hutchins (Ala. 1990) A law firm representing plaintiff in a wrongful death medical malpractice suit should have been disqualified because of one of its attorneys previous involvement as counsel for defendants. Chinese wall defense is not available under Code of Professional Responsibility. o Chinese wall refers to any set of physical and procedural boundaries intended to prevent one member of organization, such as law firm, from being exposed to information relating to a matter currently or formerly handled by one of his colleagues. CHAPTER 13 LAWYER IN LAW FIRMS AND SPECIALIZED PRACTICE AREAS Discussion Problems Rules 1.0(c), 1.10, 1.17, 2.3, 2.4, 5.1 through 5.7 1. Lasar is a senior partner in the 20-lawyer firm of Fimrite, Steele and Lasar. FSLs style of new lawyer training can best be described as sink or swim. Newly hired associates are given tasks by the firms partners and are expected to carry them out on their own, asking for help from the partners when needed. The firm is proud of the substantial responsibility given to its associates right off the bat. New associate Allen was thrilled when within one month after joining Fimrite, she was given responsibility for preparing a new will and trust arrangement for Clint, one of the firms longtime state planning clients. Lasar, as the supervising partner for Clints matters, gave Allen as example of the firms standard estate package to use as a model for her assignment. Allen was concerned about several provisions in the model such as designating managing partner Fimrite as executor and a major banking client as trustee. However, Allen thought it best to stick with the model and not ask any questions that might make her look bad or be tagged as a troublemaker. Has Allen acted properly in this matter? NO i. Rule 5.2 Responsibilities of a Subordinate Lawyer (a) A lawyer is bound by the Rules of Professional Conduct notwithstanding that the lawyer acted at the direction of another person (b)A subordinate lawyer does not violate the Rules of Professional Conduct if that lawyer acts in accordance with a supervisory lawyers reasonable resolution of an arguable question of professional duty. (c) Neuremburg defense will not work- still have an obligation under rule 5.2 (d)She should have gone to her supervisor, asked for Lasars informed opinion, and used it 86 | P a g e

What are Lasars ethical obligations in the matter? i. Rule 5.1 Responsibilities of Partners, Managers, and Supervisory Lawyers (a) A partner in a law firm, and a lawyer who individually or together with other lawyers possesses comparable managerial authority in a law firm, shall make reasonable efforts to ensure that the firm has in effect measures giving reasonable assurance that all lawyers in the firm conform to the Rules of Professional Conduct. (b)A lawyer having direct supervisory authority over another lawyer shall make reasonable efforts to ensure that the other lawyer conforms to the Rules of Professional Conduct. (c) 5.1(c) he is responsible, he and she can both be disciplined, for her direct violation, when he directed her to do this (d)Firms avoid this by having: 1) Conflict checks 2) Ethics committee Should the Fimrite firm be held accountable in any way? It cannot be because the rules to not apply to firms, just the individuals What if Allen, instead of just carrying out the assignment, expressed her concerns about the firms practices to the state agency that oversees fiduciary practices? As a result, the agency launched an investigation of Fimrite Steeles practices. When the firms partners learned of Allens role in informing the agency, she was immediately fired. Does Allen have any recourse against the firm for her firing? Only if a claim can be established without breaching attorney-client privilege, can a claim be brought; otherwise it needs to be dismissed 2. For the past four years, the law firm of Ayer & Alfred has been defending client Clayton Industries in a series of related products liability client Clayton Industries in a series of products liability cases in which the plaintiff seeks hundreds of millions of dollars for injuries caused by an allegedly defective Clayton product. The damage claims exceed Claytons assets ten-fold. International Bank and Trust Co. is now deciding whether to renew Claytons multi-million dollar line of credit. Clayton has asked Ayers and Alfred to prepare a candid evaluation of the products liability cases and to furnish it to Internationals loan department. What are the ethical obligations of A and A in this situation? You should tell that your client that you should not do it because you represent your client and it would be difficult for you to make a candid evaluation. You also run the risk of waiving the attorney-client privilege by revealing this information to a third party. Suggest that the firm have this evaluation done by another law firm. Rule 2.3 Evaluation for Use by Third Persons (a) A lawyer may provide an evaluation of a matter affecting a client for the use of someone other than the client if: the lawyer reasonably believes that making the evaluation is 87 | P a g e

compatible with other aspects of the lawyers relationship with the client (b)When the lawyer knows or reasonably should know that the evaluation is likely to affect the clients interests materially and adversely, the lawyer shall not provide the evaluation unless the client gives informed consent (c) Except as disclosure is authorized in connection with a report of an evaluation, information relating to the evaluation is otherwise protected by Rule 1.6. 3. Lawyer Acting like a 3rd partya- 2.4(a)- shall tell unrepresented lawyers to inform parties that they are not representing them b- NO c- yesd- NOTE- not the same party- & so long as it is not substantially related to the same matter that she heard while being a mediator NOTE- Rule 1.7- comment 29 and 30?...how to CYA while recruiting clients etc. 1.8(a) getting involved in related business matters 4. Law clerk should decline to work on this matter, and the firm should also be disqualified if the lawyer decides to work on it (if he has worked on it before), b/c its too uncertain and that judge could possibly end up hearing the case Its ok for the firm to take the case so long as they screen the law clerk from having contact with the case 5. Firm splitting up- can departing attys take their former clients with them? Joint letter goes out announcing that some are leaving and some attys are staying, so that gives an opportunity for the client to choose who he wants to go with 5.6(a) prohibits non-compete agreements b/c they restrict the practicing of law 6. Atty overlooked an affirmative defense that would have shielded him from liability and client sues atty for malpractice client wants atty to decline all cases that are not within his expertise for the next two years, in order to protect other people and to make an example of him. Under 5.6(a) this is a restriction of his right to practice law, and so is not ok. 7. Lawyer advertising in 20 years

Judicial Conduct1. 2. cant testify under rule_____ unless he is subpoenad He shouldnt pressure the bank, but he could call him and introduce them together 88 | P a g e

Rule 2.9? law clerk wants info for research for a water pollution case that he 3. rule 3.10 she cant.judge cant also practice law, sometimes judges can be part time though- ex. Municiapl courtshoover b. under 3.9 she cant arbitrate- but she can mediate b/c it is a judicial function c. there is an exception under rule 3.___? For religious,.. 4. Rule ______-allows but if litigation is involved, he shouldnt sit on the board b. 3.7(a)2 Cant actually raise or solicit money, but can plan fundraising campaigns but he can supervise c. d. Rule 3.6? yes, they can accept it, but thet have to report this income under 3.13 e. is it ok b/c he would have to recuse himself anyways if Anthony ever had to come to court, so the fishing pole wouldnt really affect this b/c they were already good friends f. rule 3.8-> cant serve in any fiduciary capacity except for family members 5. Judge wouldnt be disquilfied for attending the funeral b.c judges go to funerals all the timeunless he was close to the parties regardeless of going to the funeral b. 2.10 she should recuse herself b/c her public stmts, if he doesnt, they should file a writ of mandamus to get her off that case. c. 2.3 NO should recuse b.c he has a clear personal interest in how this turns out Aetna v. Levoy- US supreme court reversed Alabama ruling on the grounds of Due process b/c Alabama 5-4 swing vote judge was in the middle of the same type case with his insurance company, and so the us Suprme court reversed under DP right to fair trial d. no reason to assume bias or prejudice on the part of a judge just be looking at him e. we dont have enough facts about the casewe dont know how firivlous the objections are, or how competent the counsel is etc.this is the kind of motion youd domotion for new trial f. question is whether his interest is token or de minimus? Or whether it is substantial.if it is substantial, then he should def recuse himself 8.3(b) lawyers have a duty to report misconduct by judges report to judicial inquiry committee Judges also have a respooinsibilty to report attys about misconduct report to Bar asscoaition Judiciary inquiry committee- 1 appellate judge- 1 circuit, 1 district, 2 lay people, and 2 memebrs of the bar. Inquiries are confidential Elections of judges- we now elect our judges, since 1849 89 | P a g e

1819 constitution- jusdges were appointed by the legislature for life New Forms of Legal Practice Although traditionally lawyers practiced either as sole practitioners or in partnerships, the ABA Model Rules allow lawyers to form professional corporations as long as all the shareholders, officers, and directors are lawyers. Lawyers are also able to practice as members of group legal service plans, although the form of practice initially encountered strong opposition within the ABA. In a limited liability company (LLC), individual lawyer members are liable for their own misconduct, but may limit their personal liability while still gaining the tax treatment accorded to general partners. Limited liability partnerships (LLP) also limit a nonparticipating partner;s liability for the negligent acts of other partners. Law Firm Breakups A lawyers impending departure requires notice to the lawyers current clients under Rule 1.4. The notice can be in writing or in person or by telephone without violating Rule 7.3. Pre-departure notice by the departing lawyer to current clients may indicate the [departing] lawyers willingness and ability to continue in any current matters, but should make clear that the client has the right to decide whether to continue representation with the departing lawyer or the firm, and should not urge the client discontinue representation by the firm or disparage the firm. The departing lawyer is entitled to take copies of research memoranda, pleadings and forms prepared by the lawyer and considered the lawyers property or in the public domain and other documents the lawyer created for general use in the practice. Client files must be retained or transferred as directed by the client, although the departing lawyer can keep copies of client documents relating to the lawyers representation. Firms generally cannot prohibit their lawyers from leaving and letting clients know of their impending move. o Noncompetition clauses in partnership agreements and restrictions on the use of payments to departing partners have been used as a means of limiting competition. Their use has caused a split in the courts. Law Firm Discipline Traditionally, discipline has been a matter between the bar and its individual members. There have been proposals for law firm discipline. Entire law firms have been found liable as firms for malpractice, and for violations of regulatory regimes by administrative agencies vigorously exercising their supervisory function. Article 5 of the ABA Model Rules deals with law firm issues. FRCP 11 allows sanctioning of a law firm 90 | P a g e

General Dynamics Corp. v. Superior Court (Cal. 1994), p.305 Conclusion There is no reason inherent in the nature of an attorneys role as in-house counsel to a corporation that in itself precludes the maintenance of a retaliatory discharge claim, provided it can be established without breaching the attorney-client privilege or unduly endangering the values lying at the heart of the professional relationship. Lawyer as Problem-Solver In the culture of adversarialism, often what is important to parties may be excluded from consideration, as irrelevant or inadmissible, according to our well-worn legal principles. Pareto-optimal solutions seek the best possible solution for each party without harming the other side Collaborative lawyers attempt to solve problems as peacefully as possible, with maximum client input, and agree in advance not to pursue litigation with each other Current practices of lawyer as third-party neutral, as mediator, arbitrator, facilitator, early neutral evaluator, conciliator, fact-finder, or consensusbuilder

Professional Responsibility Brewer


August 21 Discussion questions from the book will be discussed in class 91 | P a g e

Multiple choice questions at the end of the chapter will be used for final Page 167- Answers Final- Open Rule Book 1.17 Page 156- Comment Read the rule first, then the casebook materials EXAM- part multi-state, part______ 85% grade is Final 10% paper about professionalism- due the last day of class 5% participation Malpractice suits- Violation of Rules of Ethics is not actionable in itself..Plaintiff would have to show that the lawyer breached the given std of care in that specific location etc. 6-5-578- Evidence of attempts to comply is admissible only on the defendants defense of showing that he was trying to comply with the rules. 1.6a- confidentiality- lawyer/client privilege August 23 Chapter 1 1) Rule 4.4(b)- inadvertently or mistakenly receiving emails that you shouldnt get. See comment 3- Dont use the document, return it unread, destroy it etc. 2) Rule 114(b)- can file a petition for a conservatorship on behalf of Dena, even though you represent Samueleven though there is a technically a conflict and confidential info etc. Brewer says this is an example of something that the rules permit you to dohowever, it would be very tacky to actually do. Rule 1.7 Conflict of Interest Rule 1.7(a)(2)- theres a risk that the representation of one client will be.by the relationship, etc Rule 1.6 Confidentiality- Cant disclose info regarding the representation of the client 3) Rule 2.1- we should advise our clients not only about the law, but other considerations that are relevant.i.e. moral, social, economic etc..counsel him Rule 116- If the client tells him to keep his mouth shut, what are the implications to the atty?.....the lawyer is guilty as well when he has this informationhe shouldve withdrawn if the representation of this client shall violate the law etc. Rule 1.2(d)- lawyer shall not assist a client in conduct that the lawyer knows is criminal or fraudulent.do we have a duty to disclose this??? No, not under rule 1.6(b)however, under 1.6(b) does give you the option to disclose this info if the lawyers services have been used to procure this etc.but you dont have to BUTthen do you have the duty to pay the insurance back for the overage amount? Yes, b/c he was overpaid, but how does he return the money w/o telling the insurance company the clients name? He gets his own atty to pay the insurance 92 | P a g e

from your trust account to their trust account, but dont disclose any information about your client or his overpayment August 28 4 requirements to Practice law in Alabama 1) must be 19 2) must be a graduate to an ABA approved law school- Miles and Bham School of Law are an exception ( you can take the bar as many times as you want.we used to have a statute limiting it to 5 timesbut it was struck down) 3) Pass the bar exam..Diploma privilege until the mid 70s..all you had to do was gradate from university of Alabama and then you got a certificate to practice law.we dont have reciprocity here 4) Good Moral character---Subjective requirement 3 attys Letter of Rec. stating that you have good moral character 3 phase application process 1) Law school application 2) Law student registration 3) 3rd year bar app to take the bar Practicing law is a privilege, so the burden is on the applicant to prove that he is fit to practice law Felonies are usually always automatically disqualified Crimes of moral turpitude are usually disqualified.Alabama bar association president was convicted of income tax evasion, plead nolo contondre, was suspending for a little bit, but didnt get disbarred We have n integrated barwhen you pass the bar, you are automatically are a member of the Alabama barso its automatic $300 per year dues to be a member Must to certain amount of CLE hours Dues and interest paid goes to the Alabama Bars programming etc. Do we have to join the local bar association where we practice? NO..Bham is voluntaryBham bar has ~ 4000 members Do we have to join the ABA? NOit is also voluntary, but it is the law school authority etc.

In re deBartolo deemed not good character and fitness to practice law He took the bar.and passed.and then a year later they came back and said that he didnt have the character and fitness to practice law ..but court said he had the right to a reapply, and he should not be barred for life, jus like when an atty gets disbarred, it shouldnt usually be forever 1) lied about high school attendance 2) failed to disclose addresses 93 | P a g e

3) 200-400 parking tickets (this was on his application) 4) Presented himself as a police officer Application process encourages candor and disclosure In re Mountain Adoption- attorney got the best deal for a couple who wanted to adopt, shopped around and retained the richer of the two couples He was gonna charge first couple for $500 and the 2nd couple $17k.tons of conflicts of interest.He was disbarred In re Holmay Attys forgery and notarization of clients signature on documents disclosed to opposing counsel warranted him being suspended for 30days His client was filing for divorce.the court distinguished this case from others, and called this case a case of first impression..b/c in this case, the atty DID NOT have permission from the client to forge her name Rule 5.5- Unauthorized Practice of Law Rule 8.1 Bar Admission and Disciplinary matters a) not knowingly give false info. b) Not fail to disclose fact Rule 8.2 Judicial and Legal Officials a) an atty shall not knowingly make a false stmt that an atty knows to be false, and with a reckless disregard as to its truth or falsity of a judge, legal officer, candidate for election or appointment to legal office b) an atty who is a candidate shall comply with these rules too Rule 8.3- reporting Professional Misconduct a) a lawyer has a duty to report another lawyer when he knows that the other guys conduct is in violation of rules b) same w/a judge. c) N/A to info that falls under Rule 1.6 Rule 8.4- Misconduct a) cant violate the rules of professional conduct..or assist another.or through acts of another b) commit a criminal act that reflects adversely on attys honesty, trustworthiness, or fitness as a lawyer c) conduct involving dishonesty, fraud, deceit or misrepresentation d) conduct that is prejudicial to the administration of justice e) state or imply the ability to improperly influence a government official f) knowingly assist a judge in conduct that is violative of rules Rule 8.5 Disciplinary Authority; Choice of Law 94 | P a g e

a) Lawyer admitted to a practice in a jurisdiction is subject to the laws of that state, regardless of where the misconduct occurs b) Choice of Law.. August 29 Patent bar- must have pass patent bar exam Rule 5.5(a)- can only practice law in states where you are licensedbut there are ways that you do this.reciprocity, take their bar exam Most common way Pro Hac Vice- for this term onlyfor one time only, in that state Rule 5.5(b) multi-jurisdictional practice4 ways that you can do this 1) 2) Ex- Pro hac vice 3) ADR proceedings 4) Rule 7.5(b)- you can set up a firm in another state, operate through their office etc. Rule Rule Rule Rule 8.4(b) 8.3(a)- must be a whistleblower 8.3- savings clause doesnt exist when you have a confidentiality duty under 1.6

September 4 General Rule- Lawyers are not public utilities.a lawyer may reject work for any reason that suits her.subject to important exceptions Attorneys Oath 1) to support the constitution of Alabama and the US 2) maintain respect due to courts of justice and judicial office 3) counsel or maintain such actions, proceedings or defenses only as appear to him or her legal counsel or just, except the defense of a person charged with a public offense Rule 1.16 Rule 1.18 Rule 3.1 Rule 6.1- Pro Bono Work- Any legal work we provide for persons of limited means that wouldnt be able to represent themselves etc. Rule 6.2 Bothwell v. Republic Tobacco Co. 95 | P a g e

(U.S.D.C. of Neb. 1995) p.55 Rule 116(a) there are certain situations where we must decline employment, and there are some cases that when you are already are engaged and you must quit working on it 1) when it will violate rules of professional conduct conflict of interest under Rule 1.7 or 1.9 attorney is a witness under Rule 3.7 We know our client will commit purgery 3.3(a)(4) We are not competent to handle the casenever done a criminal case etc. under Rule 1.1 2) Physical or mental handicap prevents the lawyer from adequately hadling a case a. Example- blind man cant see evidence b. Some attys cant try cases b/c of emotional problems Suppose we are already in the case, and one of these conditions comes up.what do we do? September 6 We may withdraw our services as an atty when: 1) it will not adversely affect a clients interest 2) ..crime or fraud 3) used our services to perpetrate a crime or fraud 4) client insists on taking action that we consider absolutely repugnant Rule 1.16 If he doesnt pay your feeyou can give him notice and them withdraw from the case Unreasonable burden on the attycan be financial or time related.clients too high maintenance Rule 1.16(b) when you are in litigation in the case, you must get leave of court to withdraw, for both permissive withdrawal under B or mandatory withdrawal under A(conflict of interest etc.)all you have to do is file a petition to withdraw Ruskin v. Rogers Client wanted to discharge lawyer during the trial, but the court wouldnt let him b/c it would be extremely disruptive to the litigation When an atty has the right to withdraw (no wrongful etc.) is the atty entitled to the fee for the time he has put it thus far Rosenburg v. Levin 10k fixed fe 96 | P a g e

Plus 50% of.$600k Settled w/new lawyer for $500k Quantum Meruit- $55,000 Court gave him $5000 Quantum Meruit recovery basis, but cant go over the amount contracted for.the lesser of QM and contract amount recovery Noreens case Simon spent 100 hrs on the case, plaintiff fired him w/o cause (if she had had cause,he couldnt have recovered anything).She got a new atty who settled for $13k.new aty has $1k expense= 12k net recovery First lawyers fees were $6,000 = QM..Is it fair that The plaintiff ends up getting less than her atty.5k? Is there a conflict of interest if the atty or the conflict is anonymous?? Client has a rt to discharge the firm AND to continue to be represented by the individual lawyer when he left the firm b/c he was anonymouse, and so could recover his feesbut his firm could only get QM Kriegsman v Kriegsman It was a very involved Domestic Case, she hired the firm w/ a $1000 retainer fee, and it ended up being a moster case, and so the atty wanted to withdraw from the case b/c she couldnt pay anymore.. PLAINTIFF didnt want then to withdraw from the case. Ct says that an atty may have a just and reasonable cause to withdarw from the case, unless the client consents

September 11 September 13 Rule 7.4 Exceptions- when person contacted is also another lawyer or where there is a family member, a close friend, or someone you already have a client-relatioship with Peel case pg 98- holdinfg oneself out 7.3(b) cant make contact no matter who it is with, if the atty knows that the person doesnt want to be solicited.or if solicitation involves ___________ 7.3(d)- prepaid legal services plan or services You arent soliciting, the plan is soliciting them..some jobs offer pre-paid legal services September 18 97 | P a g e

FeesIndefinite factor- fixed or contigent fees Flat fee of $1750 is prob. Reasonabl. No, under 1.5b, she is not subject to disciplinary actions for entering into this agreement via an oral contract.just preferably in writing ALL contigent fees MUST be in writing Cant have a contingent fee in a divorce case September 20 Rule 115- atys must have a trust account, that the money inside is the clients.but an atty can deposit his own money at the beginning to get it started If you are on a contigent feeyou have to inform clients that you are getting money out etc. as needed 1.15(e)- if we pocess funds that we and the client have an interest, we should maintain those funds separately unrtil the case is resolved IOLTA- interest on lawyers trust accounts Cant be on a contigent fee for criminal clientsthis is to protect attys 1.5(e) division of fees b/t attys from diff firms 1) 2) client agrees 3) total fee is reasonable 2 situations when this comes up 1) referralsgenerally 1/3 fee 2) asscoaite w/another firm.joint work.50% Spilt

Review: ***Tab Rule Book LT room 117 98 | P a g e

Rule 1.1 Rule 1.2(a), (d) Rule 1.3 Rule 1.5(a), (b), (c), (e) Rule 1.6, especially (b) exceptions (permissive) revisions to the rule will not be on the test o Attorney client privilege is a rule of evidence Rule 1.7, note 1.7(a)(2) o Liability insurer and insured conflicts Rule 1.8(a), (b), (c), (d), (e), (f), (g), (h), (i), (j), (k) make notes as to where imputed qualification applies and does not apply Rule 1.9(a), (b), (c) Rule 1.10(a) Rule 1.11 Rule 1.12 Rule 1.13(a), (b), (c) Rule 1.14 exception to the conflicts rule Rule 1.15 safekeeping of property Rule 1.16 declining or terminating representation (a) mandatory (b) permissive (c) motion for leave to withdraw in a litigated matter Rule 1.18 duties to prospective clients Rule 2.1 other factors relevant to clients position Rule 2.3 evaluation for use of third party Rule 3.1 prohibition on taking frivolous position Rule 3.3 candor toward the tribunal remedial measures ex parte / heightened duty Rule 3.4 fairness to all parties Rule 3.5 duty to the tribunal Rule 3.6 statements outside of court (b) statements permissible (c) safe harbor provision Rule 3.7 prohibition on lawyer advocating where lawyer is a witness and the exceptions n- disqualification is not imputed unless arises from 1.7 or 1.9 if testimony will be adverse to client then there is a 1.7 conflict Rule 3.8 special requirements of prosecutors Rule 3.9 role in non-adjudicative proceedings Rule 4.1 lawyers must be truthful Rule 4.2 lawyers communications with persons represented by counsel prohibited Rule 4.3 Rule 4.4 respect rights of third persons especially pay attention to (b) notify sender (returing would probably comply as well) Rule 5.1 role of supervising attorneys Rule 5.2 Rule 5.3 supervising non-lawyer assistants Rule 5.4(c)

99 | P a g e

Rule 5.5 unauthorized practice of law pro hac vice Rule 5.6 restrictions on the right to practice Rule 6.1 obligation for pro bono service Rule 6.2 responsibility when appointed by the court Rule 7.1 advertising cannot be false or misleading Rule 7.2 means of advertising Rule 7.3 solicitation Rule 8.3 requirement for reporting professional misconduct Rule 8.4 general issue of misconduct 1.0 competance 1.2(a) (d) 1.3 1.5(a) (b) (c) how to calculate fess, statement to client of remittance, (d) no contingentcy fees 9c) division of fees between attys, client agrees in writing, total fee is refundable 1.6(a) confidentiality rule, unless client consents, or discloses under 1.6(b) (b) is permissible, the rule says may, not must AND should be underlined to prevent such injury, provided laywyer was used in that connection 1.7(a) conflicts 1.7(b) exceoptsions look at comment 15 under 1.7(b) on conscionability look at insurer issues 1.8 specific conflicts(a) business conflicts Note what you have to do to get rid of the conflicts (b) prohibition (c) substabtial gift, preparing instrument (d) prohibition on literary rights (e)financial assistance form atty to client (f) third party compensation and reguirements (g) prohibits aggregate settlements (h) limitation on prospective limitation of attys liability

(i) problems w/attys aquirign settling interest in settlement value, cont. fee can be in property as well 100 | P a g e

(j) sexual relationships, only ok if they started before she was s client (k) imputes subsection of a-1 to all member of the firm 1.9(a) conflicts with former clients (b) prob representing person in sam or substantiall related matter where pervioius law firm represented adverse party (c) prob use of infor about a former cleitn to disadvantage 1.10(a) government employees sub 1.9(c) firm is also disqualified unless they have screened, and apportioned part of the fee therefrom notive given to agency so it can monitor for compliance with rile (d) currect government employee, returns to provate practice applies to law clerks as well 1.12(a) (B) MAY not disqualifiy under (a)provided.. 1.13(a) (b) duty when atty knows that employee of organization is actinf in a manner that would bring rep on org atty must take reasonable steps (c) atty given a pass to reveal confidential info , other under 1.6(a) to procted org. (d) wavier in (c) doesnt apply to internal investigation (c) if discharged in complying with rule 1.13 should notify highest person in power (f) consult org if they need advice 1.14 dimished capacityMAY take stped to protect clinet 1.15(a) safekeeping of vlients property (c) provides that trust account should be used for adcances, (d) promptly notify client and del. 1.16(a) when rep is prob must withdraw if (b) must wioth draw under 1.2(d) to avoid being associated .in most situations as (c) if matter in litigation leave of ct must be obntained 1.18 prospective clients 2.1 Lawyer as an advisor can refer to legal questions and issues, moral, economic etc 2.3 evaluations for use by 3rd parties 2.4- 3rd parties neutral may be arb, med. 3.1 asserfinf meritorious claims and defenses

101 | P a g e

3.3(a) candor disclose legal auth advers from juris and directly advers perjuered testonmony(b) remedial measures oif criminal 3.4 fainess 3.5 duty of atty to ct 5.4 5.6 restrictions on the practice (B) as part of settlemt agreemtn should agree to an agreement 6.1 duty to do pro bono work 6.2 duty to accept ct appoints 3 times you cant accpt 1) taking the case would violate rule of ethic 2) TOO FINANCIALLY BURDENSOM 3) too repugnant to. 7.2 advertising must give the name and address of at least one atty in the firm responsible in the firm 7.3 solicitation prohibition on soliticiatign in person, on the phone, or live chat etc if recorded message must give idslaimer at beg of message, or have it on the outsolde of an envelope if mailed 7.5 law firms-> letterheads etc 8.3(a) req us to report attys violating rules 8.3(b)same duty with judges 8.4 violations, catch-all

1/0- competence 1.2 (a), (d_ 1.3 1.5 (a) (b) (c) (how to calc. fee, statement to client of remittance), (d) (no contingent), (e) division of fees bw. lawyers, client agrees in writing, total fee is reasonable. 1.6(a) confidentiality rule, unless client consents, or disclosures under 1.6(b), (b) permissive (may), AND should be underlined, to prevent such injury. . . provide lawyer was used in that connection, 102 | P a g e

1.7(a)- conflict, 1.7(b) exceptions, look at comment 15 under 1.7(b) on consentablity. Look at insurer issues. 1.8 specific conflicts (a) business conflicts, note what you have to do to get rid of conflict, (b) prohibition (c) substantial gift, preparing instrument, (d) prohibition on literary rights, (e) financial assistance to a client by lawyer, (f) third party compensation and requirements (g) prohibits aggrigate settlement, (h) limitation on prospective limiation of lawyers liability, (i) proh, lawyers acquiring interest in settlement value, cont fee can be in prop as well as $ (J) sexual relationships, (k) imputes susbsections of a-I to all members of lawyers firm 1.9 (a) conflicts with former clients, (b) proh. rep person in same or sub related matter where previous law firm represented adverse party. . . (c) proh. use of info about a former client to disadvantage 1.10 (a) 1.7 and 1.9 imputees to all members, disqual not imp in other rules except 1.8(k), (b) 1.11 (a) govt employees, sub to 1.9(c), firm is also disqualified unless screening and apportioned no part of fee therefrom, notice given to agency so it can monitor for compliance with rule (d) current govt employee, reverses to private practice applies to law clerks as well 1.12 (a) , (b) may not disqualify under (a) provided. . . 1.13 (a) (b) duty when lawyer knows that ee of org is acting in a manner that would bring rep on org. lawyer must take reasonable steps, (c) lawyer given a pass to reveal info (otherwise conf under 1.6(a)) to protect org. (d) waiver in (c) doesn't apply to internal investigation, (e) if discharged in complying with 1.13, should notify highest org authority, (f) consit of org that may see advice, 1.14diminished capacitymay- take steps to protect client 1.15 (a) safekeeping of clients property, (c) provides that trust account should be used for advances, (d) promptly notify client and del. 1.16(a) when rep. is proh., MUST with drawl if. . . (b) (B says must withdraw under 1.2(d) to avoid being associated. . . in most all situations as ) (c) if matter is in litigation, leave of court must be obtained 1.18 prospective clients, 2.1- lawyer as adviser, can refer to legal question and issues, also moral, economic, etc. 103 | P a g e

2.3 evaluations for use by third parties 2.4 third party neutral, may be arb, med. 3.1 asserting of meritorious claims and defenses 3.3 (a) candor, disclose legal auth adverse, from jur, and directly adverse; perjured testimony (b) remedial measures of criminal and fraudulent conduct, (c) duties under this rule trump 1.6 (conf. rule), these duties continue to conc of proceeding, (d) ex parte, duty to dis to court all relevant facts, even those adverse to C 3.4 fairness, 3.5 (a) duty of lawyer to court, proh. imp. inf. on judge juror, etc, proh. ex parte communications, (c) proh. contact with jurors if proh. by rules (No AL rules) 3.6 (a) proh. extra jud. statement that is intended to prej proceeding, (b) list of permitted statements, (c) safe harbor provision, right ot answer a prej statement that was made by another party 3.7(a) proh. rep of person at trial where lawyer is a nec. witness, except. . . (b) dis. is not imputed to other members of lawyers firm unless conflict under 1.7 or 1.9, we defined this in class if the test would be adverse to the client, creating a COI. RULE ___ -- no duty to disclose relevant facts 4.2 proh. conversations rep by counsel, comment 7exceptionorgs rep by counsel, not proh. except set out in comment 7, Note- this exception does not work there any more, they are fair game for comm.. 4.3- unrep. persons, we should disclose our relationship to matter 4.4(a) (b) missent documents, shall notify the sender (send them back B says) 5.1 5.2 sub lawyers in law firm 5.3 5.4 prof ind o f lawyers, (a) we should not share legal fees with non lawyers (c) should not allow third parties to reg how we handle a matter (this relates to insured, insurance), (d) 5.5- unauth. practice of lawhave to desg. who is admitted in particular jur. 104 | P a g e

5.6 (a) non compete agreement proh, (b) no settlement agreements to restrict practice, also cannot ask to prohibit 6.1 6.2 ct appointments, three examples of where we should ask ct to decline, 1. 2. unreasonable financial burden, 3. cause so repug. it would affect or ability to rep. 7.1- adv cannot be false or misleading 7.2means of adv/ 7.3 solicitation, MR more restrictive, in person or live time only proh., specific requirements for paper solicitation, records must be kept. don't get conf. with other state rules. 7.5law firms, letterheads 8.3 (a) report lawyers in viol of rules (b) report judges, 8.4 generally misconduct rules listed (catch all for violations)
1/0- competence 1.2 (a), (d_ 1.3 1.5 (a) (b) (c) (how to calc. fee, statement to client of remittance), (d) (no contingent), (e) division of fees bw. lawyers, client agrees in writing, total fee is reasonable. 1.6(a) confidentiality rule, unless client consents, or disclosures under 1.6(b), (b) permissive (may), AND should be underlined, to prevent such injury. . . provide lawyer was used in that connection, 1.7(a)- conflict, 1.7(b) exceptions, look at comment 15 under 1.7(b) on consentablity. Look at insurer issues. 1.8 specific conflicts (a) business conflicts, note what you have to do to get rid of conflict, (b) prohibition (c) substantial gift, preparing instrument, (d) prohibition on literary rights, (e) financial assistance to a client by lawyer, (f) third party compensation and requirements (g) prohibits aggrigate settlement, (h) limitation on prospective limiation of lawyers liability, (i) proh, lawyers acquiring interest in settlement value, cont fee can be in prop as well as $ (J) sexual relationships, (k) imputes susbsections of a-I to all members of lawyers firm 1.9 (a) conflicts with former clients, (b) proh. rep person in same or sub related matter where previous law firm represented adverse party. . . (c) proh. use of info about a former client to disadvantage

105 | P a g e

1.10 (a) 1.7 and 1.9 imputees to all members, disqual not imp in other rules except 1.8(k), (b) 1.11 (a) govt employees, sub to 1.9(c), firm is also disqualified unless screening and apportioned no part of fee therefrom, notice given to agency so it can monitor for compliance with rule (d) current govt employee, reverses to private practice applies to law clerks as well 1.12 (a) , (b) may not disqualify under (a) provided. . . 1.13 (a) (b) duty when lawyer knows that ee of org is acting in a manner that would bring rep on org. lawyer must take reasonable steps, (c) lawyer given a pass to reveal info (otherwise conf under 1.6(a)) to protect org. (d) waiver in (c) doesn't apply to internal investigation, (e) if discharged in complying with 1.13, should notify highest org authority, (f) consit of org that may see advice, 1.14diminished capacitymay- take steps to protect client 1.15 (a) safekeeping of clients property, (c) provides that trust account should be used for advances, (d) promptly notify client and del. 1.16(a) when rep. is proh., MUST with drawl if. . . (b) (B says must withdraw under 1.2(d) to avoid being associated. . . in most all situations as ) (c) if matter is in litigation, leave of court must be obtained 1.18 prospective clients, 2.1- lawyer as adviser, can refer to legal question and issues, also moral, economic, etc. 2.3 evaluations for use by third parties 2.4 third party neutral, may be arb, med. 3.1 asserting of meritorious claims and defenses 3.3 (a) candor, disclose legal auth adverse, from jur, and directly adverse; perjured testimony (b) remedial measures of criminal and fraudulent conduct, (c) duties under this rule trump 1.6 (conf. rule), these duties continue to conc of proceeding, (d) ex parte, duty to dis to court all relevant facts, even those adverse to C 3.4 fairness, 3.5 (a) duty of lawyer to court, proh. imp. inf. on judge juror, etc, proh. ex parte communications, (c) proh. contact with jurors if proh. by rules (No AL rules) 3.6 (a) proh. extra jud. statement that is intended to prej proceeding, (b) list of permitted statements, (c) safe harbor provision, right ot answer a prej statement that was made by another party

106 | P a g e

3.7(a) proh. rep of person at trial where lawyer is a nec. witness, except. . . (b) dis. is not imputed to other members of lawyers firm unless conflict under 1.7 or 1.9, we defined this in class if the test would be adverse to the client, creating a COI. RULE ___ -- no duty to disclose relevant facts 4.2 proh. conversations rep by counsel, comment 7exceptionorgs rep by counsel, not proh. except set out in comment 7, Note- this exception does not work there any more, they are fair game for comm.. 4.3- unrep. persons, we should disclose our relationship to matter 4.4(a) (b) missent documents, shall notify the sender (send them back B says) 5.1 5.2 sub lawyers in law firm 5.3 5.4 prof ind o f lawyers, (a) we should not share legal fees with non lawyers (c) should not allow third parties to reg how we handle a matter (this relates to insured, insurance), (d) 5.5- unauth. practice of lawhave to desg. who is admitted in particular jur. 5.6 (a) non compete agreement proh, (b) no settlement agreements to restrict practice, also cannot ask to prohibit 6.1 6.2 ct appointments, three examples of where we should ask ct to decline, 1. 2. unreasonable financial burden, 3. cause so repug. it would affect or ability to rep. 7.1- adv cannot be false or misleading 7.2means of adv/ 7.3 solicitation, MR more restrictive, in person or live time only proh., specific requirements for paper solicitation, records must be kept. don't get conf. with other state rules. 7.5law firms, letterheads 8.3 (a) report lawyers in viol of rules (b) report judges, 8.4 generally misconduct rules listed (catch all for violations)

107 | P a g e

Вам также может понравиться